Sei sulla pagina 1di 67

Cardiology

A baby is born at term in good condition, requiring no resuscitation at delivery. He is


well after birth, breast-feeding well and is transferred to the postnatal ward with mum.
When he is reviewed at approximately 24 hours of age, he is noted to be cyanosed and
is transferred to the special care baby unit. On arrival on special care, he is cyanosed
with saturations of 79% in air, is mildly tachypnoeic with a respiratory rate of 62
breaths per minute and has a heart rate of 146 beats per minute.
On examination, his chest is clear and you can hear a loud systolic murmur.
A full blood count is performed which shows a haemoglobin of 18.9 g/dl with a white
cell count of 5.3 109 and normal platelets. Oxygen is administered via nasal cannula
which fails to improve his oxygen saturations.
Which of the following diagnoses are possible in this child? Choose three answers
only.
Ventricular septal defect (VSD)
Transposition of the great arteries
Patent ductus arteriosus (PDA)
Pulmonary stenosis
Total Anomalous Pulmonary Venous Drainage (TAPVD)
Coarctation of the aorta

Incorrect
Incorrect
Incorrect
Incorrect
Incorrect
Incorrect

Transposition of the great arteries


Pulmonary stenosis
Total Anomalous Pulmonary Venous Drainage (TAPVD)
Transposition of the great arteries is the commonest type of cyanotic heart
disease in the newborn. It usually presents in the first few days of life.
Examination is often normal but a systolic murmur may be heard if there is an
associated VSD or pulmonary stenosis or a patent ductus arteriosus (PDA).
Severe pulmonary stenosis presents in the newborn period with cyanosis and an
ejection systolic murmur. There is decreased blood flow to the lungs and early
intervention is required if there is a critical decrease in flow.
Total Anomalous Pulmonary Venous Drainage (TAPVD) is a condition in which
the pulmonary veins drain into the right atrium or its venous tributaries rather

than into the left atrium. If this abnormal drainage is obstructed, the baby can
present with heart failure or cyanosis in the newborn period. (Unobstructed
TAPVD presents later.)
A murmur may be the presenting feature of a VSD in a newborn but the infant
would not be cyanosed unless there were other abnormalities such as pulmonary
atresia which would decrease pulmonary blood flow. Coarctation of the aorta
and a PDA would likewise not present with cyanosis.
Match the following syndromes with the correct cardiac examination findings.
A Trisomy 21
B Noonans syndrome
C Edwards syndrome
D Di George syndrome
E Williams syndrome
F Pataus syndrome
G HoltOram syndrome
H Turners syndrome
Scenario 1

Incorrect
Normal S1 with fixed, widely split S2 and ejection systolic murmur
0 Your answer
G Correct answer
HoltOram syndrome
These clinical signs are consistent with a secundum ASD, which is associated
with HoltOram syndrome (absent radii and ASD
Scenario 2

Incorrect
Boy with ejection systolic murmur radiating to the neck with carotid thrill
0 Your answer
E Correct answer
Williams syndrome
Williams syndrome is caused by a deletion on chromosome 7q coding for elastin.
Typical features are stellate pattern in the iris, fish-shaped lips, cocktail-party
2

speech and mental retardation. Cardiac features include peripheral pulmonary


stenosis, VSD and ASD, but most notably supravalvular aortic stenosis whose
signs are described above. Here I have stated boy so as to avoid confusion with
Turners syndrome, which is also associated with aortic stenosis.
Scenario 3

Incorrect
A cyanosed infant with normal S1, single S2 and long ejection murmur best heard at
upper left sternal edge
0 Your answer
D Correct answer
Di George syndrome
These signs are suggestive of tetralogy of Fallot (VSD, right ventricular outflow
tract obstruction, right ventricular hypertrophy and overriding aorta), which is
the most common congenital cyanotic heart condition. The murmur is due to
turbulent flow in the right ventricular outflow tract and as the obstruction
becomes more sever the murmur becomes softer and shorter. Di George
syndrome is 22q microdeletion syndrome resulting in dysmorphogenesis of the
3rd and 4th pharyngeal pouches early in gestation and this leads to thymic and
parathyroid hypoplasia and other abnormalities including congenital heart
defects (also truncus arteriosus). Dysmorphic features include low set ears,
hypertelorism and short philtrum. Affected children can have variable degrees of
immunodeficiency.
Scenario 4

Incorrect
Normal S1 with ejection click and ejection murmur best heard at the upper left sternal
edge radiating to the back
0 Your answer
B Correct answer
Noonans syndrome
Noonans syndrome is an autosomal dominant condition whose features include
mild mental retardation, ptosis, webbed neck, shield chest and short stature. The
most common cardiac manifestation is pulmonary stenosis but PDA, VSD, ASD
and branch stenosis of the pulmonary arteries is also seen. The signs here are
consistent with pulmonary valve stenosis.

Scenario 5

Incorrect
Acyanotic infant with soft ejection systolic murmur and weak femoral pulses
0 Your answer
H Correct answer
Turners syndrome
These clinical features are suggestive of coarctation of the aorta, which can
present as a duct-dependent circulation with collapse in the neonatal period.
Older children may present with hypertension and a systolic murmur radiating
to the back. The key feature is the weak or absent femoral pulses. Turners
syndrome is monosomy of the X chromosome so only affects women. Girls can
have short stature, webbed neck, shield chest, widely spaced nipples, cubitus
valgus, congenital lymphoedema and ovarian dysgenesis. Up to 30% of affected
girls have a bicuspid aortic valve and about 15% have coarctation of the aorta.
An 8-month-old boy presents to the Accident and Emergency Department with a 2day history of runny nose and cough and today has had difficulty breathing, is
lethargic and has vomited once. He has no significant past medical history and his
immunisations are up to date.
On examination he looks unwell with delayed capillary refill of 45 s. Heart rate (HR)
180, respiratory rate (RR) 60, temperature 39 oC and he has increased work of
breathing. Auscultation of his precordium reveals normal first and second heart
sounds with a gallop audible and femoral pulses are both palpable. Chest and
abdominal examination is unremarkable.
Chest X-ray (CXR) shows cardiomegaly but lung fields are clear. A 12-lead
electrocardiogram (ECG) shows sinus tachycardia. He is becoming more lethargic and
once iv access is obtained he received a fluid bolus of 10 ml/kg saline and bloods are
taken for culture, FBC, U&E, C-reactive protein (CRP) and clotting.
What is the most likely diagnosis?
PDA
VSD
Viral myocarditis

Correct answer

Kawasakis disease
SVT
A diagnosis of viral myocarditis should be considered in a previously well child
who presents with cardiac failure and cardiomegaly on CXR. Enterovirus is the

commonest viral cause. There is a spectrum of severity with some children being
asymptomatic and others presenting in failure with dysrhythmias. Neonates may
present with fever, poor feeding, respiratory distress and poor pulses. A murmur
may be present suggesting mitral regurgitation from dilatation of the valve
annulus. A gallop rhythm may be heard. Blood tests may reveal raised
inflammatory markers, acidosis and raised cardiac enzymes. Viral serology may
be useful for definitive diagnosis. CXR will show cardiomegaly with pulmonary
congestion and electrocardiogram (ECG) may have ST- and T-wave changes.
Pericardial effusion may be revealed on echo. Treatment is supportive until
function improves. Diuretics are generally indicated to treat failure and ECMO
may be required as a bridge to cardiac transplant.
A Pulmonary stenosis (PS)
B Aortic incompetence
C Coarctation
D Ventricular septal defect
E Hypertrophic obstructive cardiomyopathy (HOCM)
F Atrioventricular septal defect (AVSD)
G Truncus arteriosus
H Mitral valve incompetence
I Patent ductus arteriosus
For each of the following children select the most likely cardiac lesion associated
with their condition. Each answer may be used once, more than once or not at
all.
Scenario 1

Incorrect
A 3-month-old baby with macroglossia, cardiomegaly, hypotonia and failure to thrive.
0 Your answer
E Correct answer
Hypertrophic obstructive cardiomyopathy (HOCM)Glycogen storage disease
type 2. This condition is fatal in the first year of life. It affects muscles, causing
cardiomegaly, HOCM, hypotonia and thickened muscles (including the tongue).
It is caused by decreased -glucosidase activity, leading to glycogen accumulation
in cardiac and striated muscles.
Scenario 2

Incorrect
An 8-year-old girl with ptosis, a high arched palate and a history of bleeding disorders
and learning difficulties.

0 Your answer
A Correct answer
Pulmonary stenosisNoonans syndrome is caused by a gene defect in
chromosome 12 (usually sporadic). Phenotypically like Turners syndrome, but
they tend to have right-sided heart defects (pulmonary stenosis, peripheral
pulmonary stenosis). Bleeding disorders and intellectual impairment are also
seen.
Scenario 3

Incorrect
A 1-year-old boy with butterfly vertebrae who is seen regularly by the hepatologist for
a history of cholestasis.
0 Your answer
A Correct answer
Pulmonary stenosis Alagilles syndrome is also known as arterio-hepatic
dysplasia. Hypoplasia of the intralobular bile ducts causes neonatal hepatic
cholestasis. Other features include butterfly vertebrae, persistent posterior
embryotoxon in the eye and pulmonary stenosis.
Scenario 4

Incorrect
A neonate with intra-uterine growth retardation (IUGR), jaundice and
hepatosplenomegaly, also has a purpuric rash on her skin.
0 Your answer
I Correct answer
Patent ductus arteriosus Congenital rubella syndrome. Other cardiac lesions
seen in this condition include peripheral PS and ventricular septal defect (VSD).
In addition to the above features, thrombocytopenia, cataracts, encephalitis,
chorioretinitis and deafness are seen.
Scenario 5

Incorrect
A 12-month-old girl with a history of neonatal tetany and recurrent infections.

0 Your answer
G Correct answer
Truncus arteriosusDi George syndrome is caused by sporadic chromosome 22q
deletion. Clinical features are: face: hypertelorism, ante-Mongoloid slant, cleft
lip and palate hypoparathyroidism: hypocalcaemia: neonatal tetany and
seizures absent thymus: cell-mediated immunodeficiency: candidiasis,
pneumonias etc cardiovascular system (CVS): tetralogy of Fallot, right-sided
aortic arch, truncus arteriosus, interrupted aortic arch.
A 3-month-old baby has a large VSD and is failing to thrive. He was born at term by
NVD and a murmur was heard at his neonatal check. Echo revealed a large VSD with
a moderate left to right shunt. He fed well and had no signs of failure on discharge at
1 week of age. His birthweight was 3.4 kg and his current weight is 4 kg. He is
admitted to the ward with respiratory distress, tachypnoea and tachycardia and is
awaiting repair of his cardiac lesion.
Which of the following are useful in management of this baby? Choose TWO
options
NG feeds

Incorrect

-blockers

Incorrect

Salbutamol nebulisers

Incorrect

Furosemide and spironolactone

Incorrect

Oxygen overnight

Incorrect

NG feeds
Furosemide and spironolactone
This child has been re-admitted in heart failure as evidenced by tachypnoea,
increased work of breathing, poor weight gain and probably sweating.
Pulmonary congestion can also cause wheezing and an enlarged liver may be
palpable on abdominal examination. Infants can be asymptomatic except for
when active, in the case of babies when feeding and symptoms may become
obvious as pulmonary vascular resistance falls.
Chest X-ray (CXR) would reveal increased cardiothoracic ratio and echo would
show the VSD, left-to-right shunt and one can assess left ventricular function
(expressed as fractional shortening percentage).
Diuretics are certainly helpful in the management of heart failure to relieve
pulmonary congestion and alleviate symptoms. Increasing caloric intake is
important and NG feeds are valuable to reduce the extra energy expended
during feeding and to support increased work of breathing. Oxygen may be
required to maintain saturations at the childs normal saturations but these will
be lower as there is mixing of blood across the VSD.

A 3-year-old girl is referred by her GP with a heart murmur. She was born at term by
NVD with no neonatal problems and has been generally well, except for a couple of
viral infections. Her immunisations are up to date and her growth is following the
25th centile.
On examination she is pink and well perfused with easily palpable peripheral pulses.
Her chest is clear and she has normal first and second heart sounds with a continuous
murmur heard best at the upper left sternal edge. The murmur does not radiate.
Abdominal examination is unremarkable. Electrocardiogram (ECG) is within normal
limits and echo reveals a small PDA with no evidence of hypertrophy.
What is the best definitive treatment option for this child?
Diuretics
Surgical ligation of PDA via thoracotomy
Device closure of PDA via catheterisation

Correct answer

No treatment required
Annual echo scan to monitor size of PDA
In utero, the ductus arteriosus serves to lead blood from the pulmonary arteries
to the aorta so bypassing the fetal lungs. The duct will usually close shortly after
birth but in some cases fails to do so and this more commonly happens in
premature babies. If the duct remains patent, depending on the size of the duct
and shunt, the infant can become symptomatic with features of heart failure and
left-to-right shunt. Spontaneous closure of the duct is rare after infancy.
Small PDAs are generally asymptomatic but are closed, as the later risks of
endocarditis and complications are present. Small PDAs can be closed by
transcatheter device closure (usually a coil device for small ducts) and this also
allows for catheter pressure measurements at the same time. The occlusion
device is placed in the duct under X-ray guidance and the main risk of the
procedure is a leak around the device but this is usually insignificant.
Diuretics can be useful in the management of PDA if the child is in failure so not
in this case. Surgical ligation is still an option and can be performed
thorascopically but catheter closure is less invasive.
You are the neonatal registrar on call and your SHO asks you to review a baby on the
postnatal ward. He is a term baby who is well with no concerns noted from either
mum or the midwives. There is no antenatal history of note but mum has recently
moved to the area and therefore there is no record of any antenatal scan results.
On a routine newborn check the baby was noted to have a loud, grade 3/6 systolic
murmur which is best heard at the upper left sternal edge. The rest of the examination
is normal with no other worrying features and the baby is feeding well.
An ECG is done which is within normal parameters for age.

The baby is reviewed by one of the cardiologists who performs an echo and makes a
diagnosis of Tetralogy of Fallot.
Which of the following are features of this condition? Choose four answers only:
Right ventricular hypertrophy
Left ventricular hypertrophy
Ventricular septal defect (VSD)
Atrial septal defect (ASD)
Right ventricular outflow tract obstruction
Left ventricular outflow tract obstruction
Overriding aorta
Co-arctation of the aorta

Incorrect
Incorrect
Incorrect
Incorrect
Incorrect
Incorrect
Incorrect
Incorrect

Right ventricular hypertrophy


Ventricular septal defect (VSD)
Right ventricular outflow tract obstruction
Overriding aorta
Tetralogy of Fallot is a morphological diagnosis with four main features. These
are: VSD, right ventricular outflow tract obstruction (eg pulmonary stenosis),
right ventricular hypertrophy and overriding aorta.
It is usually asymptomatic and may be diagnosed by the presence of a murmur.
Occasionally, in the presence of severe pulmonary stenosis, babies can present
with cyanosis, in which case early intervention is usually indicated. In older
infants and children, cyanotic spells can occur, usually precipitated by physical
activity, and these may require treatment with oxygen, morphine, -blockers and
putting the child in the knee-chest position to try and increase systemic venous
return.
Classical features of right ventricular hypertrophy on ECG and a boot-shaped
heart with oligaemic lung fields on chest X-ray are seen in older children.
You are asked to review a baby on the postnatal ward who the midwives suspect has
Downs syndrome.
There is no antenatal history of note and routine scans were normal. The baby was
born by spontaneous vaginal delivery and did not require any resuscitation at birth.
Both parents are fit and well and have three other children who are all healthy with no
medical problems.
On examination, you feel that the baby does have features of Downs syndrome, and

after a long discussion with the parents, blood is sent for karyotyping which confirms
the diagnosis. The rest of the examination is normal apart from a long, grade 3/6
systolic murmur heard all over the praecordium. All peripheral pulses are present and
equal, blood pressure is normal in all limbs and there are no signs of respiratory
distress.
You refer the baby to the cardiologists for review of the murmur.
What is the most common cardiac abnormality in babies with Downs
syndrome? Choose one answer only:
Tetralogy of Fallot
Persistent ductus arteriosus (PDA)
Atrial septal defect (ASD)
Atrioventricular septal defect (AVSD)
Transposition of the great arteries
Hypoplastic left heart syndrome

Incorrect
Incorrect
Incorrect
Incorrect
Incorrect
Incorrect

Atrioventricular septal defect (AVSD)


Up to half of all children with Downs syndrome may have an underlying
congenital cardiac defect and therefore these babies are routinely referred to
paediatric cardiologists for assessment even if no murmur is heard.
The most common cardiac defect in Downs syndrome is an atrioventricular
septal defect (previously called an endocardial cushion defect). PDA and
Tetralogy of Fallot do also occur but are far less frequent.

Incorrect
If this lesion is left untreated, what is the major long-term problem that could
result from this defect, and if present would cause cyanosis and deterioration in
clinical condition? Choose one answer only:
Heart failure
Hypertension
Pulmonary hypertension
Renal failure
Increasing size of the defect

10

Correct answer

Children with Downss syndrome and an uncorrected AVSD are at high risk of
developing Eisenmengers syndrome. This is a condition in which there is
increased pulmonary vascular resistance with the development of pulmonary
hypertension and a reversal of blood flow through the AVSD from left-to-right to
right-to-left. This occurs at an earlier age in children with large defects. Patients
develop signs of pulmonary hypertension such as cyanosis, right ventricular
hypertrophy with a parasternal heave and a loud pulmonary component to the
second heart sound.

Incorrect
In the presence of Eisenmengers syndrome and an AVSD, what is the most
appropriate treatment? Choose one answer only:
Medical management of the pulmonary hypertension
Surgery to close the defect
Pulmonary artery banding

Incorrect
Incorrect
Incorrect

Patching of the defect via a cardiac catheter

Incorrect

Medical management of the pulmonary hypertension


Once pulmonary hypertension has developed, with reversal of the shunt and
Eisenmengers syndrome, the defect is considered inoperable. Closing the AVSD
after this point will not help the patient and will result in heart failure and
further deterioration. The only treatment available is to try and improve the
symptoms through medical management.
A 3-year-old child had high fever for 2 weeks, with associated malaise and poor
appetite. He had dark urine for the past 2 days. There was no dysuria or frequency of
urination. There was no cough or runny nose or sore throat recently. There was no
joint pain or swelling and there was no skin rash. Family members were healthy.
There was no recent travel. On admission, body temperature was 39.5 oC. Blood
pressure was 100/70 mmHg. Heart rate was 110/min. Physical examination showed
mild pallor. Chest was clear. Apex was not displaced. There was no abnormal cardiac
impulse. A grade 3/6 systolic murmur was heard at the left sternal border. Liver was 2
cm below costal margin and a spleen tip was felt.
Investigations results were as follows:
White blood cell (WBC)

17 x 109/l

neutrophil

14 x 109/l

lymphocyte

2 x 109/l

haemoglobin

10 g/dl

Urea

5.3 mmol/l

creatinine

40 mol/l

11

alanine aminotransferase (ALT)

30 IU/l

aspartate aminotransferase (AST)

40 IU/l

Urinalysis: white blood cell (WBC) 2/HPF, numerous dysmorphic red blood cells
Which THREE of the following investigations were the most important?
Blood culture

Incorrect

Viral titre

Incorrect

Chest X-ray

Incorrect

Echocardiogram

Incorrect

Ultrasound scan of abdomen

Incorrect

Urine culture

Incorrect

Erythrocyte sedimentation rate (ESR)

Incorrect

Antinuclear antibody
Antistreptolysin O titre

Incorrect
Incorrect

Blood culture
Echocardiogram
Erythrocyte sedimentation rate (ESR)
The patient most likely had infective endocarditis, as suggested by prolonged
fever with malaise, anorexia, hepatosplenomegaly and anemia. There was an
underlying heart lesion manifested as a cardiac murmur, which was unlikely to
be flow murmur because of its loud intensity. Complication of renal emboli or
infarction had occurred as suggested by the presence of dysmorphic RBCs in the
urine. The most important investigation was several sets of blood culture to aid
diagnosis and to identify the infecting organisms. Echocardiogram was
important to demonstrate the presence of vegetations and the underlying heart
lesion. ESR was an important marker of inflammation to monitor response to
treatment. Viral titre was not indicated since viral infection rarely persists with
high fever for 2 weeks. Neutrophilia in this case indicated bacterial infection
rather than viral infection. Chest X-ray is an adjunct in any cardiac condition,
but is not an essential investigation in infective endocarditis. There was no sign of
chest involvement in this case. Ultrasound of the abdomen could demonstrate
hepatomegaly and splenomegaly but added little diagnostic information in this
case. Urine culture was not indicated in this case since he did not have symptoms
of urinary tract infection and there was no pyuria. Antinuclear antibody is a
screening test for suspected systemic lupus erythaematosus. It is rare in boys of
this age, though it can cause prolonged fever with hepatosplenomegaly, anemia
and glomerulonephritis. Antistreptolysin O titre serves as evidence of recent
Streptococcus pyogenes infection to aid diagnosis of post-streptococcal
glomerulonephritis or acute rheumatic fever. However, these diagnoses were
unlikely in this case. Post-streptococcal glomerulonephritis does not result in
hepatosplenomegaly or heart murmur. Acute rheumatic fever was unlikely at

12

this young age and there were not enough criteria, such as erythaema
marginatum, subcutaneous nodules, arthritis or Sydenhams chorea.
A 15-month-old girl is referred by her GP with a heart murmur. She is well and
thriving and there are no concerns over her development. She was born at term by
NVD and had no neonatal problems. She has had two chest infections in the last 5
months and received antibiotics for these. Her mother describes no episodes of
breathlessness of cyanosis.
On examination she is comfortable with no respiratory distress. Auscultation revels
normal first heart sound, fixed widely split second heart sound and a 2/6 ejection
systolic murmur. Her chest is clear and abdominal exam is unremarkable. Chest X-ray
(CXR) is within normal limits and electrocardiogram (ECG) reveals right-axis
deviation.
What is the most likely diagnosis?
Primum ASD
Secundum ASD

Correct answer

VSD
PDA
Truncus arteriosus
The most likely diagnosis is a secundum ASD. This can present as an incidental
finding or the child may have a degree of failure evident. Any increase in
pulmonary blood flow from left to right shunts can predispose to chest infections.
ASD accounts for 6% of congenital heart lesions and secundum defects occur at
the level of the foramen ovale. The signs outlined in the above case are classical
of ASD with an ejection systolic murmur caused by increased flow across the
right ventricular outflow tract because of the left to right shunt. The
electrocardiogram (ECG) in a secundum defect will show RAD due to right
ventricular hypertrophy, whereas a primum defect leads to LAD or superior axis
on ECG. These defects can be repaired in early childhood usually with an
autologous pericardial patch.
What are the complications of coarctation of the aorta repair?
Choose THREE options

13

Ischaemia of the spinal cord

Incorrect

Pulmonary hypertension

Incorrect

Systemic hypertension

Incorrect

Thromboembolism

Incorrect

Re-coarctation

Incorrect

Ischaemia of the spinal cord


Systemic hypertension
Re-coarctation
Coarctations are repaired by either end-to-end anastomosis or subclavian flap
repair. Systemic hypertension is not uncommon postoperatively but usually
resolves without long-term treatment. There is a small incidence of recoarctation (or residual constriction) and ischaemia of the spinal cord resulting
in paralysis is a rare complication (<0.5%) due to prolonged crossclamping of the
aorta.
The following were cardiac catheterisation results of a 12-year-old boy:
Blood pressure

Oxygen saturation

Right atrium

8 mmHg

80%

Right ventricle

100/15 mmHg

80%

Aorta

110/80 mmHg

85%

Pulmonary artery

50/25 mmHg

80%

Which of the following was the most likely diagnosis?


Pulmonary stenosis
Ventricular septal defect
Tetralogy of Fallot

Correct answer

Truncus arteriosus
Univentricular heart
The patient had cyanotic heart disease as evidenced by low oxygen saturation in
the aorta. So A and B were incorrect. Patients with VSD can have cyanosis if they
develop Eisenmengers complex. However, the pulmonary arterial pressure
should be much higher in that case. Patients with truncus arteriosus should have
equal pulmonary and aortic arterial pressures and oxygen saturations. Patients
with univentricular heart should have equal oxygen saturation in the pulmonary
artery and the aorta. This patient had Tetralogy of Fallot, with pulmonary
stenosis as evidenced by a lower pulmonary arterial pressure compared to right
ventricular pressure. The aorta had a slightly higher oxygen saturation
compared to the pulmonary artery due to mixing of deoxygenated blood in the
right heart and oxygenated blood in the left heart via ventricular septal defect
and overriding aorta. The right ventricular pressure was elevated due to
pulmonary stenosis and transmission of left ventricular pressure to the right
ventricle through the ventricular septal defect.
A premature infant born at 28 weeks of gestation had respiratory distress syndrome
after birth. One dose of surfactant was given and the baby was successfully extubated
on the second day of life. However, he developed a progressive shortness of breath on

14

the third day, with increasing oxygen requirement. Body temperature was 36.5 oC.
Respiratory rate was 70/min. Blood pressure was 45/35 mmHg. Heart rate was
180/min. Physical examination showed marked chest recession. Pulse was strong.
There was a systolic murmur heard at left upper sternal border. Abdomen was soft
with liver 3 cm below costal margin. Anterior fontanelle was normotensive. Chest Xray showed prominent pulmonary vascular markings.
Which THREE of the following treatments were most likely to be effective?
Dopamine

Incorrect

Furosemide

Incorrect

Antibiotics

Incorrect

Indometacin

Incorrect

Second dose of surfactant

Incorrect

Blood transfusion

Incorrect

Captopril

Incorrect

Dopamine
Furosemide
Indometacin
The deterioration in this child was likely due to patent ductus arteriosus, as
suggested by the presence of strong pulse, heart murmur and chest X-ray finding
of prominent vascular markings. The child was in heart failure with high
pulmonary blood flow and hepatomegaly. Effective treatments included fluid
restriction, diuretics, inotropic support, indometacin and surgical ligation of the
ductus. Antibiotic is only indicated for treatment of infection but infection could
not explain all the clinical features in this child. Although RDS was an important
consideration, there were no features of RDS in the chest X-ray and surfactant
was not helpful. The patient was not anemic and there was no sign of blood loss.
Therefore blood transfusion was not indicated. Besides, it could worsen the heart
failure. Although captopril is used in older children with heart failure, it is not
advisable in premature infant owing to its adverse effect on the kidneys and
hypotensive effect.
A 10-year-old boy is transferred to the general paediatric cardiology ward from
intensive care following a mitral valve replacement (prosthetic valve) for severe
mitral regurgitation. He is now 5 days post-op and so far has made an uneventful
recovery, is eating and drinking well and there are no current concerns. His only
medications at present are warfarin and captopril.
Shortly after his arrival on the ward you are called to see him as his temperature has
been measured at 38oC.
On examination he looked well; he was warm and well perfused. On auscultation, his
chest was clear and apart from a mitral valve click you cannot hear any abnormal
heart sounds or murmurs. His wound looked slightly erythematous and was quite
15

tender to palpation.
What should your immediate management include? Choose three answers only.
Full Blood Count
Chest X-ray
Antibiotics for suspected wound infection
Blood culture
Urine microscopy and culture
ECG

Incorrect
Incorrect
Incorrect
Incorrect
Incorrect
Incorrect

Full blood count


Blood culture
Urine microscopy and culture
In a child that has recently undergone cardiac surgery, especially valve
replacement with a prosthetic valve, the most important thing to exclude is
infective endocarditis. In order to diagnose infectious endocarditis, blood
cultures should be taken at the time of temperature spikes, along with
inflammatory markers, including white cell count and C-Reactive protein (CRP).
Urine should be sent for microscopy and culture since this is a relatively noninvasive screen and can be used to exclude a urinary tract infection as the cause
of the fever.
Such patients must not be started on empiric antibiotics (eg for suspected wound
infection) as this may prevent the growth of organisms in blood culture
specimens and end up in the diagnosis being missed (the course of antibiotics for
endocarditis is much longer than for other infections, hence the importance of
accurate diagnosis).
A chest X-ray is unlikely to be of value in a well patient with no respiratory signs
and an ECG is of no use in the diagnosis of infectious endocarditis.
A 3-day-old male term neonate with a diabetic mother was found pale with cool
extremities in the postnatal ward on day 3 of life. Blood pressure was 25/10 mmHg
measured in the right arm. Heart rate was 180/min. Respiratory rate was 50/min. On
examination, the capillary refill time was 4 s. Chest was clear with satisfactory breath
sounds. All peripheral pulses were weak. Cardiac apex was at the 5th intercostal space
in mid-clavicular line. There was a mild right parasternal impulse. The second heart
sound was split and there was a grade 2/6 systolic murmur at left sternal border.
Which of the following was the most likely diagnosis?

16

Coarctation of aorta

Correct answer

Tetralogy of Fallot
Ventricular septal defect (VSD)
Neonatal hypoglycaemia
Intraventricular haemorrhage
The scenario described a typical presentation of critical coarctation of aorta.
Patients with severe coarctation of aorta may present after a few days of life with
hypotension and shock due to left ventricular outflow tract obstruction with low
cardiac output after closure of the ductus arteriosus. All pulses, not only those of
the lower limbs may appear weak, although the lower limb pulses are usually
weaker and not palpable. The apex of a normal neonate should be located at the
4th intercostal space and hence an apex at the 5th intercostal space was displaced
due to heart failure with left ventricular dilatation. A mild right parasternal
impulse is normal in a neonate because the pulmonary arterial pressure is still
high at that time, which will fall gradually at around 46 weeks of life. A soft
ejection systolic murmur due to the coarctation can be heard at the left upper
sternal border. Neonate with shock can have normal, high or low respiratory
rate. Patients with Tetralogy of Fallot usually had cyanosis even if a spell occurs
that causes shock, rather than pallor in this child. The cardiac apex is not usually
displaced in case of Tetralogy of Fallot. A large ventricular septal defect may
present with progressive heart failure, but not shock as in this patient. The apex
can be displaced with prominent parasternal impulse due to pulmonary
hypertension. A pansystolic murmur can be heard, which is usually loud if the
VSD is large. Neonatal hypoglycaemia is a possible cause of shock and lethargy
in infant of diabetic mother. However, it cannot account for the heart murmur.
Massive intraventricular haemorrhage can cause shock, but rarely occurs in
term neonate and it cannot account for the heart murmur.

Incorrect
What THREE of the following investigations were the most important in this
case?
Prostaglandin E infusion
Dopamine infusion
Normal saline bolus
Intubation and mechanical ventilation
Oxygen supplement

17

Incorrect
Incorrect
Incorrect
Incorrect
Incorrect

Glucose infusion

Incorrect

Morphine infusion

Incorrect

Blood transfusion

Incorrect

Prostaglandin E infusion
Dopamine infusion
Intubation and mechanical ventilation
Prostaglandin infusion to re-open the ductus arteriosus and keep it patent is the
most important management of patient with critical coarctation of aorta. It
relieves the left ventricular outflow obstruction by shunting blood through the
ductus to the lower part of the body. Cardiac output, perfusion to lower parts of
the body including the kidney and heart failure symptom would be expected to
improve with prostaglandin infusion. Inotropic support was probably required
to augment cardiac contractility in this case especially in view of the presence of
hypotension. Since the patient had signs of shock, intubation with mechanical
ventilation to control the airway and breathing should be considered to ensure
adequate oxygen delivery. Ventilation should be continued until the child was
adequately stabilised. Normal saline bolus would worsen the heart failure in this
case. Oxygen supplement alone was probably inadequate in this case. Glucose
infusion was of secondary importance to maintain normoglycemia. It would be
more important if the patient had hypoglycaemia, which was not evidenced in
this case. Morphine infusion might be considered as sedation and pain control
after intubation and mechanical ventilation. It was of secondary importance.
Although the child looked pale, there was no evidence that he had acute blood
loss. The pallor and hypotension were the results of shock secondary to heart
failure in this case and blood transfusion might worsen the clinical course due to
volume overload. Cautious blood transfusion might be considered if the child
was later found to have significant anemia.
A baby is born at 36 weeks gestation and is being monitored on the special care baby
unit due to poor feeding. There are no risk factors for infection. You are called to
review her at 2 days of age because her heart rate on the monitor has gone up to 280
beats per minute (previously at 140150 beats per minute). She is settled and asleep at
present and the tachycardia has persisted for approximately 10 minutes on your
arrival.
On examination she looks well, is pink and well perfused with a capillary refill time
of less than 2 seconds and easily palpable good volume pulses. She is apyrexial with a
temperature of 36.9 C. There are no signs of respiratory distress and her chest is clear
with good air entry bilaterally. She has no audible murmurs on auscultation and her
abdomen is soft and non-distended with normal bowel sounds.
What should your first investigation be? Choose one of the following:
Bloods including Full Blood Count and CRP
Chest X-ray
Blood culture

18

ECG

Correct answer

Cardiac echo
Supraventricular tachycardia in a neonate is a difficult diagnosis as there are no
characteristic physical signs. An ECG can be diagnostic showing a tachycardia,
often around 300 beats per minute with narrow QRS complexes.
Blood tests looking for signs of infection should always be considered in an
unwell neonate and although useful, these would not help in making the
diagnosis in this case.
An echocardiogram or a chest X-ray are of no use in diagnosing
supraventricular tachycardia.

Incorrect
What treatment could be used? Choose any two of the following:
Lignocaine
Adenosine
Facial immersion in cold water
Atropine
Amiodarone

Incorrect
Incorrect
Incorrect
Incorrect
Incorrect

Adenosine
Facial immersion in cold water
The priority in management of supraventricular tachycardia (SVT) is
restoration of sinus rhythm. Vagal manoeuvres such as facial immersion in cold
water or applying an ice pack to the forehead can restore rhythm. More
frequently, intravenous adenosine as a rapid bolus is used and is often effective.
In an unstable patient with haemodynamic compromise, DC cardioversion may
be required.
Lignocaine is used in treatment of ventricular tachycardias.
Amiodarone can be used for maintenance prophylaxis against recurrent episodes
of SVT but is not used as an emergency treatment.
Choose the most approriate medication for each of the following cardiac
situations.
A Adenosine
19

B Lidocaine (lignocaine)
C Adrenaline (epinephrine)
D Amiodarone
E Flecainide
F Atropine
Scenario 1

Incorrect
A 1-week-old baby presents acutely unwell with respiratory distress and a heart rate
of 250 beats per minute. An ECG is done which shows a narrow complex tachycardia
and a diagnosis of supraventicular tachycardia is made.
0 Your answer
A Correct answer
Adenosine
An effective medical treatment for an acute supraventricular tachycardia (SVT)
is adenosine at a dose of 150250 g/kg given by intravenous bolus. A larger dose
is sometimes needed. Adenosine slows conduction through the AV node and also
suppresses automaticity of atrial and Purkinje tissues. If you cannot get venous
access, an alternative emergency treatment is to try and increase vagal tone
through facial immersion in cold water. Amiodarone can be used as a
maintenance treatment in order to provide prophylaxis against recurrent
episodes of SVT. It is used as the treatment of choice for small or premature
babies. Flecainide can also be used as prophylaxis against SVTs but is less useful
in the emergency setting.
Scenario 2

Incorrect
A 6-year-old child presents to A&E in acute respiratory distress. Soon after arrival,
she goes into cardiorespiratory arrest secondary to hypoxia. Resuscitation is
commenced and effective breaths are given via face mask with good chest movement.
On assessment, there is no pulse palpable and no signs of circulation are present.
0 Your answer
C Correct answer
Adrenaline (epinephrine)
Adrenaline is the first line drug in paediatric resuscitation. It is given in a dose of
10 g/kg body weight (0.1 ml/kg of 1:10 000 solution) via the intravenous or

20

intraosseous routes. Ten times this dose can be given down the endotracheal tube
if this is in situ and other access has not yet been obtained.
Scenario 3

Incorrect
A 12-year-old boy with a history of complex congenital heart disease has just
undergone open-heart surgery in order to replace a dysfunctional aortic valve. 24
hours after the operation he becomes short of breath and the ECG monitor shows a
tachycardia with wide uniform QRS complexes.
0 Your answer
B Correct answer
Lidocaine (lignocaine)
A wide complex tachycardia in the context of cardiac surgery and signs of heart
failure is likely to be a ventricular tachycardia. If the patient is conscious, iv
lignocaine is the drug of choice. In an unconscious patient or in the face of low
cardiac output, prompt synchronous cardioversion (at 0.5 to 1 joule/kg) should
be performed.
What is the most likely cause of hypertension in each of the cases described?
A Renal parenchymal disease
B Renal tumour
C Coarctation of the aorta
D Phaeochromocytoma
E Cushings syndrome
F Neuroblastoma
G Essential hypertension
H Hyperthyroidism
Scenario 1

Incorrect
A 2-year-old girl presents with a loud systolic murmur which is best heard at her back,
between her shoulder blades. She has weak femoral pulses and you are unable to
palpate the dorsalis pedis pulses on either side. The blood pressure in her right arm is
higher than that in her left.
0 Your answer
C Correct answer
Coarctation of the aorta
21

Coarctation of the aorta is an important cause of high blood pressure in children.


It is caused by a narrowing of the aorta, usually distal to the origin of the left
subclavian artery. If severe it may present with heart failure in infancy or in
older children presents with a murmur or high blood pressure. One of the most
important things to check for is the presence of femoral pulses and to look for
radiofemoral delay, which is often present in older children. There may be a
difference in blood pressure between the left and right sides.
Scenario 2

Incorrect
A 5-year-old girl has a history of frequent urinary tract infections for which she is on
antibiotic prophylaxis. A micturating cystourethrogram (MCUG) when she was a
baby showed the presence of vesico-ureteric reflux but she has had no further
investigations and no recent episodes of infection.
0 Your answer
A Correct answer
Renal parenchymal disease
Any child presenting with a UTI should be investigated to ensure there is no
vesicu-ureteric reflux present, which can lead to damage to the parenchyma of
the kidney and ultimate renal scarring and renal failure. If the kidney is
sufficiently damaged, this can cause hypertension. A DMSA scan is used to look
for the presence of renal scarring. Recurrent infections despite antibiotic
prophylaxis should lead one to consider the efficacy of the prophylactic cover
and the antibiotic sensitivities of the infecting organism.
Scenario 3

Incorrect
A 13-year-old girl presents with paroxysmal episodes of sweating, tachycardia,
headaches and high blood pressure. On CT scan a mass is seen in her right adrenal
gland.
0 Your answer
D Correct answer
Phaeochromocytoma
A phaeochromocytoma is a chatecholamine-secreting tumour which is usually
located in the adrenal glands. It causes paroxysmal release of adrenaline and
nor-adrenaline, thereby leading to episodes of tachycardia, sweating and high
blood pressure. Treatment is by surgical excision
22

Scenario 4

Incorrect
A 12-year-old girl is on long term oral steroid treatment for severe juvenile chronic
arthritis. She has recently developed symptoms of easy bruising, weight gain, acne
and a routine check reveals the presence of high blood pressure.
0 Your answer
E Correct answer
Cushings syndrome
Long-term use of oral steroids for any reason may cause Cushings syndrome
due to glucocorticoid excess. Clinical features of this include short stature, facial
and truncal obesity, red cheeks, hirsuitism, striae and easy bruising, muscle
wasting, glucose intolerance and hypertension.
A 7-month-old boy has been found to have a heart murmur. He was born at 36 weeks
gestation by emergency LSCS for foetal distress and weighed 2.3 kg. He required no
resuscitation at birth, no SCBU admission, fed well from birth and was discharged
after 5 days. Neonatal check and 6-week checks were normal and no murmur was
noted at either.
He is well and thriving and the murmur was an incidental finding when visiting the
Accident and Emergency Department for a viral illness. He is pink and well perfused,
femorals both palpable and chest clear. Heart sounds are normal and a soft 2/6 systolic
murmur is heard, best at the upper left sternal edge with no radiation. Heart rate (HR)
is 110 bpm, respiratory rate (RR) 32, sats 99% in air and blood pressure (BP) 90/40.
Chest X-ray (CXR) shows no cardiomegaly or pulmonary congestion and
electrocardiogram (ECG) is normal.
What is the most likely diagnosis?
Flow murmur

Correct answer

PS
VSD
AVSD
ASD
This is a classical history of an innocent murmur picked up as an incidental
finding in a child presenting for some other reason. This child is well, thriving
with previously normal cardiovascular examinations. The heart sounds are
normal and the murmur is systolic and soft with no radiation. The
cardiovascular examination is otherwise normal with normal oxygen saturations,

23

chest X-ray (CXR) and ECG. All these features point to an innocent murmur.
Such murmurs can become more obvious when a child is in a hyperdynamic
state, eg a febrile illness.
Up to 30% of children have an innocent murmur and flow murmurs are caused
by turbulent flow in the outflow tracts of the heart. Another type of innocent
murmur is a venous hum, which relates to turbulent flow in the head and neck
vessels. This is described as a continuous noise heard below the clavicles.
Innocent murmurs are systolic only, soft with normal heart sounds, normal
pulses, chest X-ray (CXR) and electrocardiogram (ECG) in an asymptomatic
child. It can be difficult to differentiate from mild pulmonary stenosis, which can
also be asymptomatic but has an ejection systolic murmur radiating to the back
and an ejection click can be heard.
Match the following descriptions of childrens conditions with the relevant
procedure.
A Subclavian flap repair
B Total cavo-pulmonary connection (TCPC)
C Balloon dilatation
D BlalockTaussig shunt
E Norwood stage-1 procedure
F RossKono procedure
H Mustard procedure
Scenario 1

Incorrect
A 4-month-old boy found to have coarctation of the aorta
0 Your answer
A Correct answer
Subclavian flap repair
Depending on the extent of constriction either an end-to-end anastomosis or
subclavian flap repair is attempted. Subclavian flap repairs leave the blood
supply to the left arm compromised but the incidence of re-coarctation is
practically zero.
Scenario 2

Incorrect
A 2-day-old baby with antenatally diagnosed hypoplastic left heart syndrome
0 Your answer
24

E Correct answer
Norwood stage-1 procedure
Hypoplastic left heart syndrome describes the severe underdevelopment of the
left ventricle and aortic arch and the heart is essentially univentricular. The
condition was previously inoperable but is not managed surgically with the
staged Norwood procedures (three procedures in total). The first stage of the
Norwood is performed in the neonatal period; the aortic arch is reconstructed
and a central shunt inserted.
Scenario 3

Incorrect
A neonate collapsed with critical aortic stenosis
0 Your answer
C Correct answer
Balloon dilatation
Balloon valvotomy can be performed via cardiac catheterisation and is the
management option for neonatal presentation as size is obviously an issue for
valve replacement. Complications include residual stenosis and regurgitation.
Scenario 4

Incorrect
A 2-month-old baby with tetralogy of Fallot and hypercyanotic spells
0 Your answer
D Correct answer
BlalockTaussig shunt
The surgical management of Fallots is a little controversial as some centres
perform a complete one-stage repair either in the neonatal period or in infancy
but that is not an option in the answer list! Some centres perform a two-stage
repair starting with a BlalockTaussig shunt (subclavian to pulmonary artery
conduit) to secure pulmonary circulation followed by a complete repair in early
childhood.
Scenario 5

25

Incorrect
A 1-year-old girl presenting for the first time with transposition of the great arteries
0 Your answer
H Correct answer
Mustard procedure
A Mustard procedure is an atrial switch operation and is an alternative to the
arterial switch operation for children who present later. Blood is diverted using
an atrial baffle to change the circulation from parallel to series but the right
ventricle remains the systemic ventricle and is prone to failure in early
adulthood. However, more recently children presenting late with TGA will have
pulmonary artery banding to reduce pulmonary blood flow and train the right
ventricle to become the systemic ventricle when the arterial switch operation is
performed later.
A 6-month-old girl presents to her GP with a cough. She was born at term by
emergency lower segment Caesarean section (LSCS) for failure to progress but
required no resuscitation at birth. Her weight at birth was on the 50th centile and since
then she has been following the 25th centile. Her mother has no concerns about her
development or feeding.
On examination she is happy and alert and sitting unsupported on her mothers knee.
The GP thinks that she looks cyanosed but has no respiratory distress. Her heart rate is
130 bpm and respiratory rate (RR) 30. She does have a slight cough and looks coryzal
but is apyrexial.
The GP sends her to the local hospital where her oxygen saturations are measured as
82% and a 3-cm liver edge is palpable. Her chest is clear, a soft systolic murmur
audible and she looks quite well. A chest X-ray (CXR) is performed which shows
modest cardiomegaly with some evidence of pulmonary congestion. An echo is
arranged for that day, which reveals she has transposition of the great arteries (TGA)
with a ventricular septal defect (VSD). Transfer to an appropriate cardiac centre is
arranged.
Before transfer what would you advise parents about her future management?
She will need an arterial switch operation immediately (one-stage
procedure)
The condition is inoperable at this late stage
Septostomy may be necessary
She should undergo VSD repair first
She may benefit from a two-stage procedure of pulmonary artery
(PA) band followed by switch operation

26

Correct
answer

Presentation of TGA with VSD at this age is unusual but can occur and obviously
definitive management will depend on the exact anatomy of the lesion. It sounds
like the infant is well but has an intercurrent viral illness, but may have sign of
heart failure with enlarged liver and pulmonary congestion.
Children who present after the neonatal period with TGA usually undergo a twostage procedure involving a PA band followed by an arterial switch operation
(ASO). Septostomy is not necessary if the child is relatively well with enough
mixing of blood. In TGA the right ventricle supports the systemic circulation
(aorta) and the left ventricle support the pulmonary circulation. A PA band is
applied to the pulmonary artery to train what will become the systemic
ventricle. If the muscle of that ventricle is not trained it may not be able to
support the systemic circulation. Once the ventricle is deemed to be adequate an
arterial switch operation can be performed to connect the residual volume (RV)
to the pulmonary circulation, the LV to the systemic circulation and close the
VSD. The coronary arteries must also be re-implanted on the neo-aorta.
A two-year-old girl is seen on the childrens day unit with a history of being
persistently unwell for the last 2 weeks.
She has been to see the GP twice over the last week with persistent fevers which have
responded to paracetamol but keep recurring despite this. She has had a cough and a
runny nose but these have improved slightly. However, she still has a poor appetite
and two days previously had developed a rash which is still present.
On examination, she is pink and well perfused but is pyrexial with a temperature of 40
C. She is well hydrated clinically but does have red, cracked lips. She has mild
conjunctivitis, a erythematous rash over her trunk and limbs and cervical
lymphadenopathy. Examination of her respiratory and cardiovascular systems is
normal and her abdomen is soft and non-tender with no palpable masses and normal
bowel sounds.
Some routine blood tests are done which show:
Hb

11.7

WCC

12.6

platelets

780

Na

136

4.5

urea

2.0

creatinine

59

CRP

<5

A blood culture is also sent to the lab.


What is the most likely diagnosis in this case? Choose one answer only:
Measles

27

Streptococcal cellulitis
Rubella
Kawasaki disease

Correct answer

Scarlet fever
Kawasaki disease mainly affects children between the ages of 6 months and 4
years. The cause is unknown but it presents as a widespread vasculitis involving
small and medium sized vessels. The diagnosis is made on the basis of clinical
signs and there are six major diagnositic criteria:
Fever, up to 40 C and perisisting for at least five days
Bilateral non-purulent conjunctivitis
Changes of the lips and mouth (cracking of skin, strawberry tongue, redness)
Polymorphic erythematous rash
Cervial lymphadenopathy (> 1.5 cm diameter)
Changes in the hands and feet (erythema, oedema and later desquamation.
Most of the blood results in this condition are non-specific but the platelet count
can be high.
The diagnosis is important as approximately one-third of affected children
develop coronary artery aneurysms (the risk of which can be lowered with
appropriate treatment such as iv immunoglobulin and aspirin)
The following data were obtained during a cardiac catheterisation of a baby who was
suspected of having a cardiac defect.
Site

Oxygen saturations %

Left atrium

96

Left ventricle

96

Aorta

96

Right atrium

74

Right ventricle

74

Pulmonary artery

89

Which of the following is the most likely diagnosis?


Ventricular septal defect
Pulmonary stenosis
Coarctation of the aorta
Atrial septal defect
Patent ductus arteriosus

28

Correct answer

This is a patent ductus arteriosus. A step-up of oxygen saturation is seen between


the right ventricle and the pulmonary artery, suggesting that oxygenated blood
has been shunted from left to right at this point. (Also remember that an increase
in pressure between two sites suggests a stenosis upstream.)

Incorrect
Which of the following conditions are associated with this defect? TWO choices
only.
Congenital rubella syndrome

Incorrect

Turners syndrome

Incorrect

Noonans syndrome

Incorrect

Maternal systemic lupus erythematosus (SLE)

Incorrect

Williams syndrome

Incorrect

Marfans syndrome

Incorrect

Homocystinuria

Incorrect

Downs syndrome

Incorrect

Congenital rubella syndrome


Downs syndrome
Turners syndrome is associated with coarctation and aortic stenosis, while
Noonans syndrome is associated with pulmonary stenosis and peripheral
pulmonary stenosis. Supravalvular aortic stenosis and pulmonary stenosis are
seen in Williams syndrome. Marfans syndrome is associated with aortic
incompetence and aortic dissection.
A 12-year-old girl is brought to the Accident and Emergency Department after
collapsing at home while revising for exams. On arrival she is alert, heart rate (HR)
110, respiratory rate (RR) 18, Sats 99% and blood pressure (BP) 115/72. She cannot
remember what happened and had no warning before the episode. Her parents
witnessed the episode and say that she just blacked out and fell to the floor, cutting
her head in the process and had no abnormal movements. She has had a couple of
episodes in the past year of feeling dizzy and faint but has not collapsed like this. She
is otherwise well with no past medical history of note and there is no significant
family history.
Examination is unremarkable including full neurological exam. Electrocardiogram
(ECG) shows sinus rhythm, normal axis, ST segments and T waves with QTc of 0.5 s.
What is the most likely diagnosis?
RomanoWard syndrome

29

Atrial tachycardia
Aortic stenosis
Prolonged QT syndrome

Correct answer

Hypertrophic obstructive cardiomyopathy (HOCM)


This is a history of cardiac syncope with no prodrome and the fact that she
sustained an injury while falling supports a blackout episode (as opposed to
gently fainting to the floor!). The previous episodes of near-syncope could also be
related to this episode. The electrocardiogram (ECG) finding of corrected QT
measurement of 0.5 s is prolonged (should be < 0.44 s) and supports the diagnosis
of prolonged QT syndrome. This often presents later in childhood and can be
provoked by exercise, stress or emotion. Forms can be genetic (AR, AD (Romano
ward), Brugada syndrome) or associated with congenital deafness (Jervell
LangNielsen syndrome). Electrolyte abnormalities should be ruled out.
Prolonged QT interval predisposes to ventricular tachyarrhythmias, which are
the cause of sudden death; torsades de pointes are particularly common in this
disease. Treatment involves -blockade and insertion of an implantable
defibrillator. If children present in VT or VF they should be treated according to
the APLS protocols.
Aortic stenosis can present with syncope but there would be abnormal findings
on cardiovascular examination. HOCM may present with displaced apex beat
and a heart murmur but one may see electrocardiogram (ECG) changes
consistent with hypertrophy.
You are asked to review a 2-hour-old baby who was born by emergency LSCS for
fetal distress. Apgar scores were 71, 95, 910 and baby required two bag and mask
breaths. The baby has remained tachypnoeic since birth with some respiratory distress
and has not been able to feed yet. There are no risk factors for infection.
Heart rate (HR) is 230 bpm, respiratory rate (RR) 68 and Sats 95% in air. Temperature
is 37.3 oC. On examination the baby has intercostal and subcostal recession and no
murmurs audible. A 2-cm liver edge is palpable.
The baby is transferred to NICU for monitoring. Chest X-ray (CXR) shows no
cardiomegaly or pulmonary plethora. A 12-lead electrocardiogram (ECG) reveals a
narrow complex tachycardia at rate of 240 bpm.
Which of the following are useful in the management of neonatal
supraventricular tachycardia (SVT)?

30

Icepacks to face

Incorrect

Verapamil

Incorrect

Adenosine iv

Incorrect

Propranolol iv

Incorrect

Cardioversion dc

Incorrect

Icepacks to face
Adenosine iv
Cardioversion dc
SVT can be difficult to diagnose in neonates as the symptoms are similar to other
neonatal conditions but electrocardiogram (ECG) should be diagnostic. In older
children it can present with palpitations, shortness of breath or even syncope. In
neonates the SVT is usually a re-entrant meaning the electrical activity goes
down through the atrio-ventricular node and then passes retrogradely up the
accessory pathway.
Investigations should include a 12-lead ECG; check electrolytes (potassium,
calcium and magnesium abnormalities can affect heart rhythm) and an Echo to
rule out structural heart disease (particularly Ebsteins anomaly and
cardiomyopathy, both associated with tachyarrhythmia).
Management is aimed at restoring sinus rhythm and depends on the clinical state
of the infant. If the baby is not haemodynamically compromised then vagal
manoeuvres can be tried. In babies this can be placing ice packs on the face for
brief periods of time (the diving reflex). Adenosine is useful to determine the
underlying pathway and is useful in terminating arrhythmia if an accessory
pathway is present. Adenosine should be used under continuous
electrocardiogram (ECG) monitoring and the trace marked when adenosine has
been given. Dosing starts at 0.05 mg/kg and can be increased incrementally to
0.25 mg/kg. The drug should be given into a large vein if possible and flushed
quickly as the half-life is very short. It is not unusual for the SVT to return once
the adenosine has worn off. If the baby is unstable then they should be intubated
and ventilated and synchronised DC cardioversion used (0.052 J/kg).
Amiodarone is another anti-arrhythmic drug that can be used but requires
loading doses. Maintenance treatment can be amiodarone or flecainide but most
children can stop maintenance after 1 year of age, as 90% will have no further
attacks. Verapamil is not used as it can cause profound hypotension.
Match the following maternal disorders and teratogens with the correct heart
diseases.
A Rubella in pregnancy
B Lithium
C SLE
D Phenytoin
E Fetal alcohol syndrome
F HHV6 in pregnancy
G Aspirin
Scenario 1

Incorrect
Patent ductus arteriosus (PDA)

31

0 Your answer
A Correct answer
Rubella in pregnancy
Rubella in pregnancy causes congenital heart lesions in up to 35% of babies born
to affected mothers. The two conditions associated with rubella are PDA and
peripheral pulmonary stenosis and it is infection before 8 weeks gestation, which
causes heart disease. Other features of congenital rubella include cataracts and
deafness.
Scenario 2

Incorrect
Atrial septal defect
0 Your answer
E Correct answer
Fetal alcohol syndrome
Up to 70% of children with fetal alcohol syndrome have cardiac lesions, which
are variable. They include ASD, VSD, tetralogy of Fallot and coarctation of the
aorta. Children also have a typical facies with long, featureless philtrum and
developmental delay.
Scenario 3

Incorrect
Aortic stenosis
0 Your answer
D Correct answer
Phenytoin
Phenytoin is teratogenic and causes the fetal hydantoin syndrome. Apart from
craniofacial abnormalities and nail abnormalities cardiac lesions include aortic
stenosis, pulmonary stenosis, coarctation of the aorta and PDA.
Scenario 4

Incorrect
32

Congenital heart block


0 Your answer
C Correct answer
SLE
SLE and Sjgrens syndrome can affect the fetus causing congenital heart block.
The cause is anti-Ro antibodies, which cross the placenta. Babies can be severely
affected and born hydropic requiring urgent treatment. Management will
depend on the stability of the baby; if the baby is haemodynamically unstable a
chronotropic agent (isoprenaline) may be required. Implantable pacemakers are
available but insertion is limited by the size of the infant.
Scenario 5

Incorrect
Ebsteins anomaly
0 Your answer
B Correct answer
Lithium
Lithium used to treat maternal bipolar disorder can cause Ebsteins anomaly.
The anatomical lesion is an abnormal tricuspid valve whose leaflets are stuck to
the ventricular wall. There is an ASD and the inlet of the right ventricle becomes
arterialised. Infants are cyanosed and often do not thrive. They are prone to
SVTs. The characteristic murmur is a long systolic murmur and also a diastolic
murmur. Management depends on severity and age at presentation but neonates
may require nitric oxide for pulmonary vasodilation or the ductus arteriosus
opening. Neonatal presentation has a poor prognosis.
A 12-hour-old baby is found on the postnatal ward to be grunting and dusky. He was
born at term by normal vaginal delivery (NVD) after an uneventful pregnancy with
normal antenatal scans. On examination he has increased work of breathing with
subcostal and intercostal recession, nasal flaring and grunting. He has a systolic
murmur and 2-cm liver edge palpable. Capillary refill time is 3 s and he is alert. His
heart rate (HR) is 190 and respiratory rate (RR) 68. Oxygen saturations are measured
as 72%. He is transferred to the neonatal intensive care unit (NICU) where his
perfusion becomes poor and respiratory effort reduced.
What are the immediate next steps in the management of this baby? (Choose
TWO).
Intubation and conventional ventilation

33

Incorrect

High-flow oxygen

Incorrect

High-frequency oscillation ventilation (HFOV)

Incorrect

Inhaled nitric oxide

Incorrect

Prostaglandin infusion

Incorrect

Inotropic support

Incorrect

Intubation and conventional ventilation


Prostaglandin infusion
This is a classical presentation of a baby with a duct-dependent cardiac lesion
becoming symptomatic as the duct closes shortly after birth. Here the symptoms
are respiratory distress, poor perfusion and cyanosis and cyanotic congenital
heart disease must be considered despite the history of normal antenatal scans.
The initial management of this neonate begins with airway, breathing and
circulation as any resuscitation would. It does not matter what the exact
diagnosis is, as the management will be essentially the same, but duct-dependent
lesions should be considered as this might influence commencing a prostin
infusion. As the baby has poor perfusion and reduced respiratory effort it would
be appropriate to intubate and ventilate the baby (airway and breathing) but
HFOV would not be necessary at this stage. Prostin infusion may help to keep
open a closing duct and buy time to allow full diagnosis and transfer to
appropriate cardiac centre. Inhaled nitric oxide is not indicated in the initial
management but may be of use if pulmonary hypertension is suspected.
Inotropic support may be indicated but this would depend on blood pressure
measurements and perfusion once ventilated. Theoretically high-flow oxygen is
not advised in duct-dependent lesions as oxygen stimulates ductal tissue to
contract and close the duct but often babies need FiO2 >21% to maintain
saturations in the 70s.
A 3-year-old boy is seen in the Accident and Emergency Department with a 6-day
history of fever and irritability. He saw his GP 3 days previously, who prescribed a
course of oral antibiotics for tonsillitis but his fever has continued and he has become
more miserable. Today his mother noticed his hands were red and the skin was
starting to peel and on direct questioning she remembers his eyes being quite red a
few days earlier.
His temperature is 39.6 oC, heart rate (HR) 140, respiratory rate (RR) 36, Sats 97%
with immediate capillary refill. His chest is clear with normal first and second heart
sounds and no murmurs. Abdominal examination is unremarkable. His throat is red
with slightly enlarged tonsils but no pus and a 2-cm cervical lymph node palpable on
the left side of his neck. He has a blanching maculopapular rash over his trunk.
Blood results are as follows:
Hb

11.0

WCC

21.1

34

NQ

16.8

platelets (Plt)

500

clotting

normal

electrolytes and renal function

normal

aspartate aminotransferase (AST)

40

C-reactive protein (CRP)

198

chest X-ray (CXR) and electrocardiogram (ECG)

unremarkable

What is the most likely diagnosis?


Rheumatic fever
Kawasakis disease

Correct answer

Infective endocarditis
HenochSchnlein purpura
Streptococcal infection
The most likely diagnosis is Kawasakis disease, a vasculitis illness also known as
mucocutaneous lymph node syndrome. It is characterised by fever lasting more
than 5 days, polymorphous rash, non-suppurative conjunctivitis, cervical
lymphadenopathy and swelling and desquamation of the hands and feet.
Children are also typically very miserable and irritable. Blood results can show
raised white cell count (WCC) and C-reactive protein (CRP), transaminitis and
after the second week (usually) rising platelet count which can reach >1000.
Treatment consists of intravenous immunoglobulin (either one dose or two
depending upon response) and aspirin. Initially the aspirin dose is antiinflammatory (30100 mg/kg per day) which then falls to 25 mg/kg per day once
the fever has settled. This is continued depending on echo findings.
Echo scan should be performed in the 2nd or 3rd week of the illness and is to
look for coronary artery aneurysms, one of the main complications of
Kawasakis disease and now the leading cause of acquired heart disease in
children in the western world. Patients who are unresponsive to intravenous
immune globulin (IVIG) (temperature does not settle) are more likely to form
aneurysms. If the echo is abnormal repeated scans would be required to
determine if the lesion is static. Most aneurysms will regress within 2 years.
An alternative diagnosis could be streptococcal infection (toxic shock syndrome)
with tonsillitis and desquamation with raised inflammatory marker and white
cell count, but this child does fit the criteria for Kawasakis disease.
A 10-year-old boy is seen in A&E with a minor injury to his ankle sustained while
playing football. On doing his observations, the nurse looking after him notes his
blood pressure to be 180/100 and you are asked to review him.
On history, he is otherwise well. He takes no regular medications and is normally

35

active and plays football twice a week. His weight and height are within normal
limits. There is no other past medical history of note but mum does mention that he
had a urinary tract infection when he was younger which cleared up with antibiotics.
On examination, he looks well, his chest and cardiovascular examinations are normal
and his abdomen is soft and non-tender with no palpable masses. You re-check his
blood pressure and this is still raised with a systolic reading of 170 in both arms.
What is the most likely cause of the high blood pressure in this child? Choose one
answer only:
Anxiety
Phaeochromocytoma
Neuroblastoma
Renal scarring

Correct answer

Co-arctation of the aorta


The most likely cause of high blood pressure in a child is secondary to another
underlying condition. It is very important to re-check the readings, ensuring the
correct cuff size is used and that the blood pressure is checked in both arms.
In a child with a history of a urinary tract infection, the most likely cause would
be renal parenchymal disease, eg scarring secondary to reflux nephropathy.
Therefore it would be important to check urea and electrolytes and perform a
renal ultrasound and a DMSA scan in order to exclude this as the cause. Other
investigations looking for other causes of secondary hypertension (eg
catecholamine excess secondary to phaeochromocytoma or steroid excess due to
Cushings syndrome) should also be performed.
Hypertension in a child can be asymptomatic or may present with proteinuria,
headaches, seizures or cardiac failure. It is important to do a full examination
looking specifically for abdominal masses and evidence of papilloedema in any
child presenting with hypertension.
A 10-year-old girl is admitted to the general paediatric ward with a fever and
polyarthritis. She has had swelling of her ankles and wrists over the last week (the
swelling is starting to improve) and yesterday developed redness and swelling of both
knees as well.
On examination, there is limited movement of her knee and ankle joints due to pain.
There is evidence of pericarditis on cardiovascular exam in the form of a pericardial
friction rub and she has a temperature of 39 C. There is no rash at present though her
mum does describe a rash which was present 2 weeks before which sounds like a
macular papular rash with a darker border to the lesions.
You do some routine blood tests which show:
36

Hb

11.6

WCC

16.3

platelets

458

CRP

67

ESR

is raised at 42

On further questioning, her mother mentions that they moved to the UK from
Zimbabwe 3 years previously and that she had a similar illness shortly before they
moved but this resolved spontaneously. They went back to Zimbabwe to visit relatives
approximately 6 weeks ago.
What is the most common long-term cardiac problem resulting from this
condition? Choose one answer only.
Aortic stenosis
Mitral regurgitation
Tricuspid stenosis
Mitral stenosis

Correct answer

Constrictive pericarditis
The features described would fit with a diagnosis of acute rheumatic fever. This
is caused by an abnormal immune response to a preceding Group A betahaemolytic streptococcus (Streptococcus pyogenes) infection. It is more common
in developing countries. Rheumatic fever is diagnosed based on the finding of
clinical criteria (Jones criteria). Major criteria include pancarditis, polyarthritis,
erythema marginatum, Sydenhams chorea and subcutaneous nodules. Minor
criteria include fever, polyarthralgia, history of rheumatic fever, raised acute
phase reactants (ESR and CRP) and prolonged PR interval on ECG. Two major
or one major and two minor criteria are required to make the diagnosis as well
as evidence of previous Group A strep infection.
The most common long-term problem is mitral stenosis resulting from scarring
and fibrosis of the valve. More rarely, aortic and tricuspid valve disease can
occur. Valve disease can develop up to 5 to 10 years after an attack of acute
rheumatic fever. Symptoms do not usually develop until later in life. Prophylaxis
against further attacks of rheumatic fever is recommended after an initial
episode (penicillin).
An anxious mother brought a 4-week-old baby girl to a general practitioner
complaining that she had been feeding poorly in the past 1 week. Her antenatal and
perinatal course was uneventful. Pre-discharge examination on day 2 of life was
entirely normal. On physical examination, the body temperature was 39 oC and the
child had profuse sweating. Respiratory rate was 60/min. Heart rate was 180/min.
Breath sounds were normal but there were crepitations heard occasionally. All
peripheral pulses were easily palpable. Apex beat was located at the 6th intercostal
37

space in mid-axillary line. There was a strong parasternal impulse. The second heart
sound was split and a grade 2/6 systolic murmur was heard at left sternal border.
Which of the following was the most likely diagnosis?
Tetralogy of Fallot
Ventricular septal defect (VSD)

Correct answer

Patent ductus arteriosus


Myocarditis
Pneumonia
Ventricular septal defect was the most compatible diagnosis with the clinical
description. A normal newborn examination could not exclude a congenital heart
disease. The heart murmur of ventricular septal defect and heart failure
symptoms might not be obvious in the first few days of life when the pulmonary
arterial pressure was still high. With progressive fall of pulmonary arterial
pressure, heart failure symptoms would become more obvious and the child
might have poor feeding, tachypnoea, displaced apex, basal lung crepitations and
hepatomegaly. The strong parasternal impulse could be caused by pulmonary
hypertension due to high pulmonary blood flow from left to right shunt through
the large VSD. Fever in a child with congenital heart disease raised the
possibility of infective endocarditis, which should be considered in this patient. A
strong parasternal impulse is also present in patients with Tetralogy of Fallot.
But it usually causes cyanosis without heart failure symptoms and signs, unless it
is the type of pink Fallot with large VSD and mild pulmonary stenosis, which
has symptoms and signs sometimes indistinguishable from a large VSD. But pink
Fallot is certainly rarer than a large VSD. Patent ductus arteriosus can present
as heart failure and give easily palpable pulses. However, the murmur is usually
continuous unless it is a small-sized ductus, which may only have murmur
detectable in the systolic phase. However, a small ductus should not present with
heart failure symptoms and signs. Acute myocarditis could cause heart failure
symptoms and signs as in this patient, as well as the fever. However, it could not
account for the heart murmur. Similarly, while pneumonia might cause fever and
tachypnoea and basal crepitations, it could not account for the displaced cardiac
apex and murmur. Nevertheless, pneumonia can complicate most congenital
heart diseases with haemodynamic disturbances, such as a large VSD.

Incorrect
What THREE of the following investigations were the most important in this
case?
Echocardiogram
Cardiac catheterisation
Chest X-ray

38

Incorrect
Incorrect
Incorrect

Blood culture
Virus study
Chromosomal analysis

Incorrect
Incorrect
Incorrect

Echocardiogram
Chest X-ray
Blood culture
Echocardiogram was the single most important investigation to define the
underlying structural heart defects. Chest X-ray was an adjunct to aid diagnosis
of congenital heart disease. It was of higher importance in this case since the
child had fever with lung crepitations. The child might have pneumonia
complicating the ventricular septal defect. A chest X-ray also helped to assess the
degree of left to right shunt by visualizing the amount of pulmonary vasculature.
Blood culture was an important investigation of possible infective endocarditis in
this case. Cardiac catheterisation was not urgently needed in this case. It might
be performed if operative repair of the VSD was considered at a later stage after
fever settled. Virus study is more important in cases of myocarditis, although the
fever in this patient might have been caused by a virus. This was of lower
importance here, since viral identification probably would not help the
management in this setting. Chromosomal analysis is worth considering in
patients with congenital heart disease. But it is of secondary importance for
patient management and frequently has poor yield if there is no other associated
dysmorphic features or other malformations.
Choose the most common genetic association for each of the conditions
described:
A Downs syndrome
B Williams syndrome
C Di George syndrome
D Noonans syndrome
E Marfans syndrome
F PraderWilli syndrome

Scenario 1

Incorrect
A baby is born with an antenatal diagnosis of a chromosome abnormality, initially
suspected by the finding of increased nuchal translucency on ultrasound. She has a
complete AVSD (atrioventricular septal defect) on cardiac echo.
0 Your answer
39

A Correct answer
Downs syndrome
Downs syndrome is often associated with congenital heart disease and there is a
30% incidence of heart defects in these babies. The most common condition is a
complete AVSD (also called an endocardial cushion defect).
Scenario 2

Incorrect
A baby has a truncus arteriosus diagnosed antenatally. After birth, he is noted to have
lymphopenia, hypocalcaemia and a cleft palate as well as characteristic facial
features. FISH testing of chromosomes shows a deletion on chromosome 22.
0 Your answer
C Correct answer
Di George syndrome
Di George syndrome is caused by a deletion on chromosome 22 (22q11.2
deletion). It results in abnormal development of the third and fourth pharyngeal
pouches during intrauterine life. Cardiac associations can include trucus
arteriosus, interrupted aortic arch, Tetralogy of Fallot and VSD. Hypocalcaemia
can occur and usually presents in the first few days or weeks of life. There is
thymic hypoplasia leading to lymphopaenia. Characteristic facial features
include hypertelorism, low-set ears, antimongoloid slant to eyes, micrognathia,
carp-shaped mouth and cleft palate.
Scenario 3

Incorrect
A 14 year old boy presents with classical clinical and echo features of mitral valve
prolapse with mitral regurgitation which was diagnosed after a murmur was heard by
his GP. He is tall, as are other members of his family and his only medical history of
note is a spontaneous pneumothorax one year ago.
0 Your answer
E Correct answer
Marfans syndrome
Marfans syndrome is an autosomal dominant condition which results in a defect
in the formation of collagen. Patients are tall and may have features of joint
laxity, arachnodactyly, pectus excavatum, kyphoscoliosis and high arched palate.
40

Lens dislocation and myopia may occur. Cardiac associations include aortic and
mitral valve regurgitation as well as a dilated aortic root with the possibility of
aortic dissection.
Which of the following conditions are associated with pulmonary stenosis?
Choose TWO options only.
Pompes disease

Incorrect

Alagilles syndrome

Incorrect

Friedreichs ataxia

Incorrect

Turners syndrome

Incorrect

Marfans syndrome

Incorrect

Homocystinuria

Incorrect

Fetal alcohol syndrome

Incorrect

Williamss syndrome

Incorrect

Alagilles syndrome
Williamss syndrome
Pulmonary stenosis is seen in Noonans, Williams and Alagilles syndromes.
Some other associations include:
Pompes disease and Friedreichs ataxia: hypertrophic obstructive
cardiomyopathy (HOCM)
Turners syndrome: aortic stenosis, coarctation
Marfans syndrome: aortic dissection, aortic incompetence.
A 4-month-old infant has undergone an atrioventricular septal defect (AVSD) repair
and pre-operatively was in failure with a large left to right shunt and failure to thrive.
Shortly after return from theatre his blood pressure and oxygen saturations fall
consistent with a pulmonary hypertensive crisis.
What is the management?
Sedation paralysis

Incorrect

Increase FiO2

Incorrect

Inhaled nitric oxide

Incorrect

Dopamine infusion

Incorrect

Prostaglandin infusion

Incorrect

Ventilate in air

Incorrect

Sedation paralysis
Increase FiO2
Inhaled nitric oxide

41

Children who have a large left to right shunt are at risk of pulmonary
hypertension postoperatively, so a 4-month-old baby with AVSD, who was
clinically in failure and not thriving, is at risk. Pulmonary hypertensive crises are
caused by a change in pulmonary vascular resistance leading to reduce
pulmonary blood flow. This manifests clinically as desaturation followed by
hypotension. If the right atrial pressure was being measured this would also be
seen to rise.
If a child is ventilated they should be put into 100% oxygen and hand bagged so
they can be hyperventilated to promote pulmonary vasodilation. Sedation is also
important and paralysis can also be useful. Inhaled nitric oxide is a potent
pulmonary vasodilator and may be useful if crises persist. Pre-oxygenation in
100% oxygen and sedation before stimulation is recommended (eg before
physiotherapy or suctioning).
Although hypotension is a result of pulmonary hypertension dopamine infusion
is probably not necessary for this reason (may be needed for myocardial
dysfunction) and prostaglandin is not indicated.
A baby is born in good condition at term and requires no resuscitation at birth. There
are no antenatal concerns and he is transferred to the postnatal ward with mum. You
are called to review him at approximately 12 hours of age because he is feeding
poorly, has started grunting and according to the midwives is a strange colour.
On arrival he is slightly tachypneic with mild subcostal recessions but is not grunting
at present. He is noticeably cyanosed and therefore you immediately transfer him to
special care. His saturations are 75% in air on arrival on the special care baby unit but
they improve to 80% with 100% oxygen.
On examination, he is mildly tachypnoeic with recessions. His chest is clear. On
auscultation you can hear a systolic murmur all over the praecordium.
What should your first steps in management be? Choose two answers only.
Start iv antibiotics
Check blood glucose
Start iv fluids
Start an infusion of Prostin
Give oxygen
Do an ECG
Check blood glucose
Start an infusion of Prostin

42

Incorrect
Incorrect
Incorrect
Incorrect
Incorrect
Incorrect

In any unwell newborn with grunting and respiratory signs, it is important to


check a blood glucose. This can be done on a capillary sample and is a relatively
straightforward thing to correct if abnormal. It should therefore be included in
your initial assessment.
In this baby there are signs of cyanosis which does not correct significantly with
oxygen treatment. You are suspecting a cardiac condition in this case. Babies who
become cyanosed shortly after birth due to cardiac causes possibly have a ductdependent cardiac lesion and may worsen significantly once the patent ductus
arteriosus closes (thereby effectively decreasing blood flow to the lungs further).
In these babies it is important to try and keep the duct open and this is done with
the use of prostaglandin E (Prostin).
An ECG is usually normal and unhelpful in this setting. Antibiotics and iv fluids
may be indicated but are usually unnecessary and there is the risk of causing
fluid overload in such babies. However, iv 10% dextrose is often used, especially
in babies with low blood sugars or who are unwell. Oxygen is of no significant
benefit in these babies.

Incorrect
A chest X-ray is performed which shows clear lung fields and a normal heart shadow
with possibly a slightly narrowed upper mediastinum.
What is the most likely diagnosis?
Hypoplastic left heart dyndrome
Ventricular septal defect (VSD)
Tetralogy of Fallot
Coarctation of the aorta
Transposition of the great arteries

Correct answer

Transposition of the great arteries is the commonest type of cyanotic heart


disease in the newborn and the most likely diagnosis in a cyanosed neonate with
narrowing of the upper mediastinum on chest x-ray. A murmur suggests a
coexistent VSD or pulmonary stenosis or may be due to a patent ductus
arteriosus. The diagnosis is made by echocardiography which shows the aorta
arising from the right ventricle and the pulmonary artery from the left.
Neither a VSD nor coarctation of the aorta would present with cyanosis in a
neonate but would more commonly present with heart failure later in infancy.
Tetralogy of Fallot usually presents in infancy and causes cyanosis if there is
severe right ventricular outflow tract obstruction. Chest X-ray here shows a

43

boot-shaped heart with pulmonary oligaemia.


Hypoplastic left heart syndrome presents with heart failure rather than cyanosis.
A 9-month-old baby with tetralogy of Fallot is admitted to hospital with increased
hypercyanotic spells.
How do you manage spelling? Choose FOUR options.
100% oxygen

Incorrect

Dopamine infusion

Incorrect

-blockers

Incorrect

Draw legs up to chest

Incorrect

Morphine

Incorrect

100% oxygen
-blockers
Draw legs up to chest
Morphine
Tetralogy of Fallot is a complex cyanotic congenital heart disease comprising of
VSD, right ventricular outflow tract obstruction (RVOTO), overriding aorta and
right ventricular hypertrophy. The condition results in reduced blood flow to the
lungs and presentation is variable depending on the lesion and size of the
pulmonary arteries. If pulmonary arteries are small then infants can present
early with cyanosis. Correction involves either a definitive repair or a two-stage
procedure; BlalockTaussig shunt early to ensure adequate pulmonary blood
flow followed by definitive repair (VSD closed and transannular patch repair of
the RVOTO). Postoperative complications include right ventricular dysfunction
and dysrhythmias.
Spelling is caused by spasm of the infundibular muscle which causes shunting of
blood from right to left across the VSD. This leads to increased cyanosis and
hypotension. Management of spells involves 100% oxygen, drawing the legs up to
the chest to increase systemic venous return and -blockers, generally
propranolol. Morphine can be useful to reduce any pain and calm the baby as
cyanosis worsens as the baby becomes more upset.
Match the following clinical scenarios with the relevant arrhythmia.
A Junctional ectopic tachycardia (JET)
B Sinus arrhythmia
C WolffParkinsonWhite syndrome
D First degree heart block
E SVT
F Torsade de pointes
G Prolonged QT syndrome
H Complete heart block
44

Scenario 1

Incorrect
A neonate is born with a heart rate of 60 bpm but is haemodynamically stable.
Electrocardiogram (ECG) shows P waves dissociated from QRS complexes
0 Your answer
H Correct answer
Complete heart block
A heart rate of 60 bpm in a neonate is pathological but this baby is
haemodynamically stable so cardiac massage is not appropriate. If the P waves
bear no relation to QRS complexes this is complete heart block and a common
cause of congenital complete heart block is maternal SLE.
Scenario 2

Incorrect
A 14-year-old girl arrives in the Accident and Emergency Department following an
overdose. Her electrocardiogram (ECG) shows polymorphic ventricular tachycardia
with QRS complexes of varying amplitudes
0 Your answer
F Correct answer
Torsade de pointes
This is a description of torsades de pointes, a polymorphic form of VT where
complexes change in amplitude and the QRS axis twists around the isoelectric
line. It can spontaneously terminate but can degenerate into VT or VF causing
sudden death. Causes include tricyclic antidepressant overdose, hypocalcaemia,
hypomagnesaemia and prolonged QT syndrome.
Scenario 3

Incorrect
A child is being investigated for episodes of palpitations. Electrocardiogram (ECG)
reveals slightly shortened PR interval and waves
0 Your answer
C Correct answer

45

WolffParkinsonWhite syndrome
WolffParkinsonWhite syndrome is caused by an accessory pathway between
the atria and ventricles. Symptoms include episodes of SVT, palpitations, chest
pain and dizziness. Characteristic electrocardiogram (ECG) findings are a
wave (a slurring of the Q upstroke), short PR interval and wide QRS
complexes. Accessory pathways can be treated with ablation.
Scenario 4

Incorrect
A neonate is born with a heart rate of 60 bpm but is haemodynamically stable.
Electrocardiogram (ECG) shows P waves dissociated from QRS complexes
0 Your answer
H Correct answer
Complete heart block
A heart rate of 60 bpm in a neonate is pathological but this baby is
haemodynamically stable so cardiac massage is not appropriate. If the P waves
bear no relation to QRS complexes this is complete heart block and a common
cause of congenital complete heart block is maternal SLE.
Scenario 5

Incorrect
A 10-month-old boy returns from theatre following AVSD repair. After a few hours
his heart rate (HR) increases to 190 bpm and regular rhythm. A 12-lead
electrocardiogram (ECG) reveals QRS complexes of normal morphology but
dissociated from P waves (atrio-ventricular dissociation)
0 Your answer
A Correct answer
Junctional ectopic tachycardia (JET)
This is a classical history and findings of JET. It may occur following cardiac
surgery; infants at higher risk are those who have had a right ventriculotomy
such as atrio-ventricular septal defect (AVSD) repair and Fallots repair. The
rhythm is temperature sensitive, which is why keeping these children cool, can
help control the tachycardia. There is atrio-ventricular (AV) dissociation and the
ventricular rate exceeds the atrial rate (so rate generally between 170200 bpm)
with normal QRS complexes and often retrograde P waves. Management

46

involves correcting any electrolyte imbalances, cooling, temporary pacing via


pacing wires placed during the operation and amiodarone.
Select ONE condition that most closely matched the description.
A Small ventricular septal defect
B Large ventricular septal defect
C Mitral regurgitation
D Mitral stenosis
E Aortic stenosis and regurgitation
F Pulmonary stenosis and regurgitation
G Patent ductus arteriosus
H Atrial septal defect
Scenario 1

Incorrect
A 3-month-old boy presented with progressive shortness of breath for 2 months and
poor feeding. Weight gain was poor. Physical examination showed mild subcostal
recession. Chest was clear. Pulse was normal. A right ventricular impulse was felt.
Apex beat was located at the 6th intercostal space in mid-clavicular line. The second
heart sound was split and there was a grade 3/6 systolic murmur over the left sternal
border.
0 Your answer
B Correct answer
Large ventricular septal defect
The patient had symptoms of heart failure resulting from a large ventricular
septal defect. The displaced apex indicated left ventricular volume overload. The
right ventricular impulse was due to pulmonary hypertension secondary to
increased pulmonary blood flow. The murmur of VSD is typically pansystolic.
Small ventricular septal defect does not result in heart failure symptoms or
displaced apex. Mitral regurgitation can result in pulmonary venous congestion
causing respiratory distress. It also causes left ventricular volume overload with
displaced apex and a pansystolic murmur. But the murmur should be best heard
at the apex rather than the left sternal border.
Scenario 2

Incorrect
An asymptomatic 3-year-old girl was told to have a heart murmur by a general
practitioner. Physical examination showed a strong pulse. Apex was located at the 5th
intercostal space in mid-clavicular line. There was no parasternal heave. Heart sounds

47

were split and there was a grade 3/6 systolic murmur and grade 2/6 diastolic murmur
over the left sternal border.
0 Your answer
G Correct answer
Patent ductus arteriosus
The child likely had a patent ductus arteriosus, as suggested by the strong pulse.
It typically causes a continuous machine-like murmur. However, sometimes there
may be a short split between the systolic and diastolic phase, resulting in both
systolic and diastolic murmurs. Aortic stenosis and regurgitation can result in
both systolic and diastolic murmurs. However, the pulse should not be strong
because of aortic stenosis. Pulmonary stenosis and regurgitation can also result
in both systolic and diastolic murmurs. However, the pulse should be normal and
a right ventricular impulse should be felt.
Scenario 3

Incorrect
A 10-year-old child had occasional palpitations. Physical examination showed a
normal pulse. Apex beat was located at the 6th intercostal space in mid-clavicular
line. There was no parasternal heave. Heart sounds were split and there was a grade
3/6 systolic murmur over the apex.
0 Your answer
C Correct answer
Mitral regurgitation
The patient most likely had mitral regurgitation. Palpitation might be secondary
to left atrial enlargement and associated atrial fibrillation. The apex was
displaced due to left ventricular volume overload. A pansystolic murmur best
heard at the apex radiating to axilla is classical.
Select ONE condition that most closely matched the description.
A Infective endocarditis
B Acute myocarditis
C Pericarditis
D Cardiac rhabdomyoma
E Acute rheumatic fever
F Chronic rheumatic heart disease
G Kawasaki disease
Scenario 1

48

Incorrect
A 4-year-old girl presented with fever for 5 days, with generalised maculopapular rash
and swollen hands and feet. Cardiac apex was located at the 5th intercostal space at
mid-clavicular line. Heart sounds were normal without murmur.
0 Your answer
G Correct answer
Kawasaki disease
Fever for 5 days with generalised rash and swollen extremities are most
compatible with Kawasaki disease. Although infective endocarditis might give
rise to rash rarely, it should not cause swollen extremities. The rash related to
acute rheumatic fever is classically erythraemia marginatum, not generalised
maculopapular rash. Acute rheumatic fever does not cause extremity swelling,
but arthritis is a feature.
Scenario 2

Incorrect
A 7-year-old boy presented with fever for 2 days, with nausea, vomiting and lethargy.
Cardiac apex was located at the 6th intercostal space at mid-clavicular line. Heart
sounds were normal without murmur.
0 Your answer
B Correct answer
Acute myocarditis
Acute myocarditis was the most likely diagnosis in this scenario. Nausea and/or
vomiting is almost a constant feature of acute myocarditis. The patient also had
evidence of heart failure as suggested by a displaced apex. Infective endocarditis
was less likely as there was no cardiac murmur or underlying congenital heart
disease. Acute pericarditis was also not likely since a 7-year-old child should be
able to complain of chest pain, which is common in case of acute pericarditis.
Acute rheumatic fever is extremely rare in this age and there was no other sign
suggesting this diagnosis.
Scenario 3

Incorrect

49

A 10-year-old girl presented with fever for 2 days and anterior chest pain and lethargy.
Cardiac apex was located at the 6th intercostal space at mid-clavicular line. Heart
sounds were muffled without murmur.
0 Your answer
C Correct answer
Pericarditis
Among the given choices, acute pericarditis was most likely to give rise to fever
with chest pain. The muffled heart sounds were due to pericardial effusion
associated with pericarditis. Acute myocarditis usually does not cause chest pain
or muffled heart sounds. Kawasaki disease may cause ischemic chest pain in the
presence of coronary artery complications such as stenosis or thrombosis of an
aneurysm. But this usually does not occur in the acute stage with short course of
fever. Chronic rheumatic heart disease and cardiac rhabdomyoma are usually
painless.
Scenario 4

Incorrect
A 9-year-old girl presented with fever for 10 days, with malaise and anorexia. Cardiac
apex was located at the 5th intercostal space at mid-clavicular line. Heart sounds were
normal and there was a systolic murmur at left sternal border.
0 Your answer
A Correct answer
Infective endocarditis
Infective endocarditis is the most likely cause of prolonged fever, malaise,
anorexia and a heart murmur due to underlying congenital heart disease. Fever
of acute myocarditis or acute pericarditis is usually of shorter duration and heart
murmur is not a feature in these conditions. Fever in Kawasaki disease can be
quite protracted but again heart murmur is uncommon.
Scenario 5

Incorrect
A 6-month-old boy presented with poor feeding for 4 weeks, with irritability. Cardiac
apex was located at the 5th intercostal space at mid-clavicular line. Heart sounds were
normal and there was a diastolic murmur at left sternal border.
0 Your answer

50

D Correct answer
Cardiac rhabdomyoma
Cardiac rhabdomyoma was the most likely diagnosis in this case. It is associated
with tuberous sclerosis and tends to occur in infancy with spontaneous resolution
in about half of the patients as they grow up. Rhabdomyomata are commonly
located near atrioventricular valve causing functional mitral or tricuspid
stenosis, leading to a diastolic murmur. Chronic rheumatic heart disease has
never been reported in such a young infant, though it is associated with heart
murmur without fever. Fever is a constant feature of Kawasaki disease. Acute
pericarditis or myocarditis does not usually cause heart murmur.
A 4-year-old boy is referred to paediatric outpatients after a GP heard a murmur
during a consultation for an upper respiratory tract infection. You review him in the
clinic.
He was born at term with no antenatal problems and routine health checks have been
normal. He is generally fit and well but suffers from occasional wheezy episodes,
especially when he has a cold. He is on no medications and has no known allergies.
There is no family history of heart disease.
On examination, he looks well and is on the 50th centiles for both height and weight.
He is pink with saturations of 99% in air. His peripheral pulses are all palpable with
normal volumes. His chest is clear but on auscultation of his heart you can hear a
murmur localised to the pulmonary area. His heart sounds are normal and his apex is
palpable in the fifth intercostal space in the midclavicular line. His abdomen is soft
and non-tender and his liver is not palpable.
You diagnose an innocent murmur.
What are the features of this type of murmur? Choose four answers only:
Systolic
Diastolic
Grade 12/6 in intensity
Varies with posture
Does not vary with posture
Radiates to other positions

51

Incorrect
Incorrect
Incorrect
Incorrect
Incorrect
Incorrect

Localised to one position

Incorrect

Harsh

Incorrect

Systolic
Grade 1-2/6 in intensity
Varies with posture
Localised to one position
Innocent murmurs are very commonly heard in children. The murmur described
above could possibly be a pulmonary flow murmur.
Innocent murmurs are always systolic. They are soft with a musical or vibratory
quality, are localised and poorly conducting, vary with posture and occur in a
child with an otherwise normal cardiovascular examination. They are present in
the presence of fever (causing a high output state) but may be heard at any time.
The ECG and chest X-ray are normal in these children. No further investigations
are usually necessary, although a cardiac echo may be done if available to
exclude other causes and to provide reassurance for parents.
A 13-year-old boy is seen in the general paediatric clinic with the complaint of being
unable to play in his football matches. He complains of shortness of breath when
running as well as chest pain during matches. His mum tells you that he has had these
symptoms on and off over the last few years but they have got worse recently and are
now interfering with his daily activities. He is otherwise well with no past medical
history of note and he is on no current medications. There is no family history of any
heart disease and he has two younger siblings who are well.
On examination, he looks well, is pink and well perfused. His chest is clear with no
audible wheeze. On cardiovascular examination, his pulses are regular with normal
volumes, he has a grade 3/6 ejection systolic murmur heard at the upper left sternal
border which seems to radiate into the neck. His abdomen is soft and non-tender with
no palpable liver.
What is the most likely cause of his symptoms? Choose one answer only:
Asthma
Aortic stenosis

Correct answer

Hypertrophic cardiomyopathy
Aortic regurgitation
Ventricular septal defect
Aortic stenosis may be an isolated lesion or may be associated with other
conditions such as mitral valve abnormalities or with co-arctation of the aorta. It
may present with heart failure in the neonatal period (babies may be dependent
on blood flow across the ductus arteriosus in order to maintain systemic blood
flow and they therefore develop heart failure as the ductus closes). Later in
childhood it may present with an asymptomatic murmur or, if severe, with
reduced effort tolerance, chest pain and syncope.

52

Signs on examination are a small volume pulse, a soft aortic component to the
second heart sound and an ejection systolic murmur, heard maximally at the
aortic area and radiating to the carotids. A carotid thrill may also be present. An
ECG may show signs of left ventricular hypertrophy.
If intervention is required, balloon or surgical valvotomy is performed. In
neonates or children requiring early treatment, aortic valve replacement is often
necessary at a later stage.
Asthma is unlikely in this child with a clear chest on examination and no past
history of asthma. The other conditions listed may all present with a murmur but
aortic stenosis is the most likely cause in this situation.
Which of the following cardiac lesions are associated with Williams syndrome?
Choose TWO answers only.
Aortic stenosis

Incorrect

Atrial septal defect

Incorrect

Ventricular septal defect

Incorrect

Hypertrophic obstructive cardiomyopathy (HOCM)

Incorrect

Aortic incompetence

Incorrect

Supravalvular aortic stenosis

Incorrect

Patent ductus arteriosus

Incorrect

Pulmonary stenosis (PS)

Incorrect

Supravalvular aortic stenosis


Pulmonary stenosis (PS)
Williams syndrome is associated with pulmonary stenosis, peripheral PS and
supravalvular aortic stenosis. Williams syndrome is caused by a microdeletion
on chromosome 7. Sufferers have characteristic facial features: prominent lips,
open mouth, blue eyes (so-called elfin facies). They present with idiopathic
hypercalcaemia in the first 2 years of life and failure to thrive. They are also
over-friendly (cocktail party personality), with learning difficulties.
Which of the following can cause a prolonged QT interval in an
electrocardiogram (ECG)? Choose ONE answer only.
Hypercalcaemia
Hyperthyroidism
Hyponatraemia
Fever
Hypocalcaemia

53

Correct answer

Causes of a prolonged QT interval include:


hypothyroidism, hypocalcaemia, hypomagnesaemia, hypothermia
severe rheumatic carditis
drugs: amiodarone, tricyclic antidepressants
congenital: Romano-Ward syndrome, Jervell-Lange-Nielson syndrome (also
associated with sensory-neural deafness).
A 12-year-old girl is admitted from A&E with a fever of 40 C and feeling generally
unwell.
She is a known cardiac patient and had a mitral valve replacement for severe mitral
regurgitation 2 months previously. Since her operation, however, she has been fine
and has been coping well at home over the month. Over the last 2 days she has
developed a runny nose and a cough and has been complaining that the wound from
her operation is quite sore. She has mild shortness of breath although she thinks this
may be improving slightly.
On examination, she is pyrexial and has mild conjunctival pallor. She is not clubbed
and there is no peripheral oedema. She has a respiratory rate of 40 breaths per minute
and scattered crackles in her chest bilaterally. Her heart sounds are normal and she has
a soft, grade 12 out of 6 systolic murmur heard maximally in the mitral area. Her
abdomen is soft and non-tender with no masses and a liver palpable 1 cm below the
costal margin. Her wound is quite red and tender to palpation. You do some routine
blood tests.
The results are as follows:
Hb

9.7

WCC

20.3

neutophils

18

platelets

564

Na

143

4.8

urea

3.5

creatinine

64

CRP

14

What should your choice of treatment at this point be? Choose one answer only:
Treat with iv antibiotics for a lower respiratory tract infection
Treat with iv antibiotics for a wound infection
Give oral antibiotics
Start empiric antibiotics for bacterial endocarditis

54

No treatment

Correct answer

Bacterial endocarditis should be considered in any patient with a prostethic


heart valve (or with congenital heart disease or previous cardiac surgeries) who
presents with a persistent fever. The risk is greatest in lesions with turbulent
blood flow or in patients with prosthetic valves. Clinical signs include pallor,
fever, changing cardiac murmurs, arthraligia or arthritis, haematuria and the
rarely seen peripheral stigmata of splinter haemorrhages, Janeway lesions and
Oslers nodes.
Echocardiography is used to look for the presence of vegetations on the cardiac
valves.
In these patients it is vitally important to establish the diagnosis before starting
on antibiotic treatment as treatment for endocarditis is a prolonged, usually six
week, course of intravenous antibiotics and it is important to establish the
identification of the infecting organism to ensure appropriate cover. Therefore
empiric antibiotics should not be started and positive culture results should be
awaited.

Incorrect
If this patient had required a dental extraction in the preceding weeks it would have
been important to provide adequate prophylaxis against endocarditis in the form of
antibiotics before the procedure.
What is the most appropriate antibiotic choice in these situations? Choose one
answer only:
Penicillin
Ciprofloxacin
Flucloxacillin
Amoxicillin
Ceftazidime
Trimethoprim

Incorrect
Incorrect
Incorrect
Incorrect
Incorrect
Incorrect

Amoxicillin
Antibiotics prophylaxis for infective endocarditis is required for most children
with unoperated or postoperative congenital heart lesions. Some children for
example, a closed PDA or repaired ASD dont need long-term prophylaxis and
therefore local guidelines and policies should be followed. Dental procedures
carry the risk of systemic bacteraemia and therefore need antibiotic cover before
the procedure. Prophylaxis is also needed for certain ENT, genito-urinary or
55

gastrointestinal procedures.
The usual recommended antibiotic is amoxicillin orally, or if allergic or had
recently received penicillin, then clindamycin. High-risk patients receive a
further dose after the procedure (local policies and recommendations should be
used).
The mother of a 3-year-old child with Tetralogy of Fallot noticed that he was very
irritable for the past 5 min and turned deeply blue. On admission, body temperature
was 37 oC. Blood pressure was difficult to be measured since the child was very
irritable. Heart rate was 180/min. Physical examination showed no significant heart
murmur.
Which THREE of the following treatments were most likely to be effective in this
case?
Oxygen supplement

Incorrect

Dopamine infusion

Incorrect

Morphine bolus

Incorrect

Bicarbonate bolus

Incorrect

ATP bolus

Incorrect

Intubation and mechanical ventilation

Incorrect

Oxygen supplement
Morphine bolus
Bicarbonate bolus
The situation described was hypercyanotic spell in-patient with Tetralogy of
Fallot. Effective treatments include a kneechest position, oxygen supplement,
propranolol, morphine, bicarbonate and systemic vasoconstrictors.
Select the most appropriate from the following options:
A Ventricular septal defect (VSD)
B Innocent murmur
C Patent ductus arteriosus (PDA)
D Aortic stenosis
E Mitral stenosis
F Aortic regurgitation
G Venous hum
Scenario 1

Incorrect
A well six-year-old girl has a short, soft (grade 1-2/6) systolic murmur which seems to
vary with respiration. It is localised to the left sternal edge with no radiation.
56

0 Your answer
B Correct answer
Innocent murmur
Innocent murmurs are also known as physiological or flow murmurs and are
commonly heard in normal children with no underlying heart disease. They are
soft, systolic, short in duration with a musical or vibratory quality. They are
localised to one area of the praecordium and tend to vary with respiration or
posture.
Scenario 2

Incorrect
A 6-month-old baby has a murmur diagnosed by his GP. It is a continuous murmur,
best heard under the left clavicle.
0 Your answer
C Correct answer
Patent ductus arteriosus
A patent ductus arteriosus (PDA) is commonly found in preterm infants but can
also be present in older children. It is a continuous murmur, best heard in the left
infraclavicular region, grade 24 out of 6 in intensity. There may be associated
features of bounding pulses and an active praecordium.
Scenario 3

Incorrect
A 6-week-old baby is referred to clinic with a murmur diagnosed by his GP on routine
check. On examination he looks well but you can feel a thrill and hear a loud, grade
4/6 systolic murmur, well localised at the lower left sternal edge.
0 Your answer
A Correct answer
Ventricular septal defect (VSD)
A VSD is the commonest form of congenital heart disease. It can present with
heart failure at around 6 weeks of age or as an asymptomatic murmur at any
age. It presents as a systolic murmur, grade 25/6 in intensity, often well localised
at the lower left sternal border. A thrill may be present. Management depends on
the size of the defect and the amount of flow between the ventricles.
57

A 6-month-old girl with tetralogy of Fallot is about to undergo repair. Of note in her
past medical history she was admitted to the Special Care Baby Unit (SCBU) for
observation, as she was jittery. Blood tests at that time revealed hypocalcaemia but
she has been well since. On examination she is alert and well perfused but looks
cyanosed. Saturations are 86% in air. She has hypertelorism, slightly low set ears and
is thriving. Auscultation reveals a 3/6 ejection systolic murmur at the right sternal
edge and there are no signs of failure.
Which of the following tests might be helpful to establish an underlying
diagnosis? Choose TWO options.
Lymphocyte subsets

Incorrect

FISH for 22q microdeletion


Order irradiated blood products

Incorrect

Airway assessment

Incorrect

Thrombophilia screen

Incorrect

Incorrect

Lymphocyte subsets
FISH for 22q microdeletion
Tetralogy of Fallot can be associated with Di George syndrome (22q
microdeletion syndrome), as can other cardiac defects such as truncus arteriosus
and double aortic arch. 22q11 microdeletion results in dysmorphogenesis of the
3rd and 4th pharyngeal pouches early in gestation and this leads to thymic and
parathyroid hypoplasia and other abnormalities including congenital heart
defects. Neonates can present with hypocalcaemia as a result of underdeveloped
parathyroid glands. Dysmorphic features include low set ears, hypertelorism and
short philtrum. The immune defect associated with Di George (affecting ~25% of
children with 22q11 deletion) is primarily a T-cell defect but the severity is
variable and in some mildly affected children can correct with time.
FISH is the genetic test for 22q microdeletion and would be valuable in preoperative work-up, as would lymphocyte subset (LSS). LSS are useful as they
guide whether leucodepleted irradiated blood products are required for surgery
to prevent graft versus host disease due to cell-mediated immunodeficiency
associated with the syndrome. Irradiated blood products can be ordered when
the results are known. There are no specific airway or thrombophilia problems
associated with Di George syndrome.
An ex-26 week premature baby is now 5 weeks old on the NICU and weighs 1.6 kg.
Her course on NICU has been complicated by an episode of NEC (not requiring
surgery) and two episodes of sepsis. She has just started tolerating enteral feeds but
remains on some TPN.
Although it has been possible to wean the ventilator settings the baby has failed
extubation twice, being unable to remain stable on CPAP. Examination reveals a pink
and well-perfused baby with a long line in situ. Heart rate (HR) is 150 bpm and Sats

58

94% in air. On auscultation of the chest one can hear transmitted sounds and a heart
murmur audible in systole and diastole. Echo scan shows a large patent ductus
arteriosus and some volume loading of the left atrium.
What treatment options are available for this baby? Choose THREE options
Surgical ligation

Incorrect

Diuretics

Incorrect

Indometacin

Incorrect

Captopril

Incorrect

Sildenafil

Incorrect

Surgical ligation
Diuretics
Indometacin
PDA is more common in premature infants and if the PDA is large with a left to
right shunt it can prevent babies weaning from the ventilator (as in this case). It
can also increase the risk of NEC and chronic lung disease. The shunt is at the
arterial level between the pulmonary artery and aorta. Abnormal ductal tissue
can result in the duct failing to contract and close, but in the majority of cases
the duct will close spontaneously. Echo is diagnostic and will give an idea of how
large the shunt is and whether the left side of the heart has become enlarged.
Management can be conservative or surgical. Fluid restriction and diuretics are
used as pulmonary blood flow is increased if the duct is large. Indometacin (a
prostaglandin synthetase inhibitor) encourages ductal constriction and is usually
given as a short course. Ibuprofen is also being used now. Sometimes surgical
ligation of the duct through a left thoracotomy is necessary to allow weaning of
ventilation.
Captopril is an angiotensin-converting enzyme (ACE) inhibitor used in some
children following complex congenital heart disease repair and sildenafil is an
oral pulmonary vasodilator. Neither drug is indicated in this case.
A 5-year-old girl with Turner syndrome is referred to the cardiology outpatient clinic
following the detection of a murmur by her GP.
She is currently well, is growing well and there are no parental concerns at present.
She has no shortness of breath on exertion, does not complain of chest pain and there
are no other symptoms of cardiac disease. She has a past history of pneumonia at the
age of 3 years, for which she required hospitalisation and iv antibiotics. Parents
remember the doctors mentioning something about a murmur at that point but no
follow-up was arranged.
On examination, she looks well and is pink with saturations of 98% in air. She has
features of Turner syndrome including short stature, wide carrying angle (cubitus
valgus) and a broad neck. There is no respiratory distress and chest is clear. All
59

peripheral pulses are palpable including the femorals and dorsalis pedis pulses. On
cardiac auscultation you can hear a grade 23 out of 6 systolic murmur which radiates
to the back between the scapulae. Her blood pressure is 90 systolic in her left arm and
110 systolic in her right arm.
What is the most likely cardiac diagnosis? Choose one answer only.
Aortic stenosis
Coarctation of the aorta

Correct answer

Mitral stenosis
Innocent murmur
Ventricular septal defect (VSD)
Coarctation of the aorta is the commonest cardiac defect in children with Turner
syndrome. It can present in infancy with cardiac failure if the coarctation is
severe or in older children can present with a heart murmur or hypertension.
The constriction usually occurs just distal to the origin of the left subclavian
artery.
The key to clinical diagnosis is the presence of weak or absent femoral pulses or
the presence of radiofemoral delay and it is important to check for this in these
patients. The classical murmur is an ejection systolic murmur best heard at the
back between the shoulder blades. There is often elevated blood pressure in the
right arm.
The chest X-ray in coarctation is usually normal but may show cardiomegaly
and increased vascularity in children with heart failure. In older children there
may be rib-notching due to the presence of collateral vessels. The ECG may
show left ventricular hypertrophy.
A 14-year-old boy suddenly fell with loss of consciousness while playing basketball
for 30 min in the morning. His classmate witnessed that he was pale and all his four
limbs were flaccid afterwards. He enjoyed good past health. His paternal uncle died
suddenly at the age of 25 years with unknown cause. On arrival to accident and
emergency department 10 min later, he regained full conscious level, his blood
pressure was 100/60 mmHg and heart rate was 100/min. Physical examination
showed regular, normal volume pulse with apical impulse. Heart sounds were normal
and there was no murmur. Twelve-lead electrocardiogram (ECG) showed sinus
rhythm. Neurological examination was normal.
Which THREE of the following investigations were the most important to
determine the underlying cause of the event?

60

Electroencephalogram (EEG)

Incorrect

Echocardiogram

Incorrect

24-hour ambulatory ECG monitoring

Incorrect

Blood glucose

Incorrect

Tilt table test

Incorrect

Erect and supine blood pressure determination

Incorrect

Computed tomography (CT) scan of the brain

Incorrect

Echocardiogram
24-hour ambulatory ECG monitoring
Blood glucose
The most likely diagnosis in this case was hypertrophic obstructive
cardiomyopathy (HOCM), as suggested by family history of sudden death and
the clinical scenario of collapse during exercise with an apical impulse on
physical examination. It is diagnosed by echocardiogram. Heart murmur may
not be obvious on clinical examination and ECG may not show features of left
ventricular hypertrophy. To exclude hypoglycaemia by haemoglucostix is
essential in the first place since it is readily correctable by giving glucose.
Hypoglycemia was possible in this case since the collapse occurred during
exercise and he had pallor after falling down. A 24-hour ECG monitoring is also
helpful for diagnosing paroxysmal cardiac arrhythmia, which can cause sudden
death that runs in family, eg, arrhythmogenic right ventricular dysplasia. Such
arrhythmia may be transient and the patient may regain consciousness when
sinus rhythm returns. EEG is helpful for investigation of seizure disorder, which
was less likely in this case because of no tonicclonic movements observed during
the period of loss of consciousness. It could be considered as a fourth
investigation if allowed by the question. Tilt table test and supine-erect blood
pressure (BP) measurements are investigations for orthostatic hypotension or
vaso-vagal syncope, which usually occurs after prolonged standing or a change of
posture from supine or sitting to erect posture and rarely occurs during exercise.
CT scan of the brain was not indicated in this case in the absence of significant
head injury or neurological deficit.
A 10-week-old baby presents with tachypnoea, poor feeding with shortness of breath
and sweating after feeds and failure to gain weight.
On examination he is pale and looks unwell. He is afebrile. He is tachypnoeic with a
respiratory rate of 80 breaths per minute and marked subcostal and sternal recessions.
He has scattered crackles in his chest bilaterally but good air entry and no audible
wheeze. On cardiovascular examination he is markedly tachycardic with a heart rate
of 280 beats per minute. His pulses are all palpable and auscultation reveals normal
heart sounds with no murmurs heard. His abdomen is soft and non-distended with his
liver palpable 4 cm below the costal margin and no palpable spleen.
Blood tests reveal:
Hb

12.6

WCC

3.5

61

platelets

345

Na

140

4.7

urea

9.8

creatinine

74

An ECG is performed which shows a narrow complex tachycardia.


What are the possible causes of this babys heart failure? Choose three answers
only:
Supraventicular re-entry tachycardia
Atrial flutter
Ventricular tachycardia
Complete heart block
Atrial ectopic tachycardia
Bundle branch block

Incorrect
Incorrect
Incorrect
Incorrect
Incorrect
Incorrect

Supraventicular re-entry tachycardia


Atrial flutter
Atrial ectopic tachycardia
The most common arrhythmia in this setting is a persistent supraventricular reentry tachycardia. Most cases are due to atrioventricular re-entry via an
accessory pathway. Babies present with poor feeding or heart failure but may
also present with cardiogenic shock, in which case rapid treatment is necessary.
The ECG shows a narrow complex tachycardia. P waves may be visible.
The priority in treatment is the restoration of sinus rhythm and adenosine is
usually used for this purpose. Vagal manoeuvres could also be tried (eg applying
an ice pack to the forehead or facial immersion in cold water). Maintenance
treatments will be necessary once the baby has been restored to sinus rhythm.
Atrial flutter may also present as a narrow complex tachycardia causing heart
failure but is rarer and usually well tolerated. Atrial ectopic tachycardia is
another narrow complex tachycardia which can lead to compromised cardiac
function.
Ventricular tachycardia and bundle branch block would show wide QRS
complexes on ECG. Complete heart block would present as a bradycardia rather
than as a tachycardia.

62

The mother of a 3-month-old infant noticed that he suddenly turned pale with
sweating during bottle-feeding. The infant was lethargic on arrival to accident and
emergency department after 20 min. Heart rate was 220/min. Blood pressure was
65/40 mmHg. Respiratory rate was 40/min. Electrocardiogram (ECG) showed a QRS
axis of 100o, with duration of 100 ms. There was no obvious P wave.
What was the best next step in the management for this child?
Close observation
Give oxygen supplement
Perform echocardiogram
DC cardioversion
Give adenosine

Correct answer

This child had narrow complex tachycardia, most likely due to paroxysmal
supraventricular tachycardia (SVT). The child was symptomatic with pallor and
lethargy. Urgent treatment rather than observation alone was required, lest the
infant might decompensate with deterioration in cardiac output and shock. The
blood pressure was normal and there was no urgency to proceed directly to DC
cardioversion. Adenosine is an effective treatment option in SVT and should be
considered. Giving oxygen alone could not revert SVT to sinus rhythm and was
useless unless the child was in shock or hypoxia. Echocardiogram was useful in
screening for underlying structural heart disease predisposing to SVT, but was
not needed in the urgent assessment and management of this patient.
Which of the following are recommended as prophylaxis for bacterial
endocarditis in children at risk undergoing dental procedures? (Choose TWO)
Augmentin

Incorrect

Septrin

Incorrect

Ciprofloxacin

Incorrect

Clindamycin

Incorrect

Amoxicillin

Incorrect

Clindamycin
Amoxicillin
The risk of endocarditis is related to turbulent flow damaging endothelium and
anything, which results in bacteraemia (dental cleaning, surgery) can lead to
seeding of vegetations on damaged endothelium. Lesions are categorised into
high-, moderate- and low-risk categories and all congenital cyanotic heart
diseases and TGA, Fallots and shunts fall into the high-risk category, as do
previous episodes of infective endocarditis and any prosthetic valves. Surgically
repaired ASD, VSD and PDAs fall into the moderate risk category. Exceptions to
requiring prophylaxis include ASD, which have only required suturing, not patch

63

repair. Prophylaxis is also recommended for one year after catheter device
closure of lesions.
Amoxicillin is recommended (<5 750 mg, 5-10 1.5 g, > 10 3 g) or clindamycin in
penicillin-sensitive patients. Azithromycin can also be used for penicillin-sensitive
patients. Antibiotics are given 1 hour before surgery or dental treatment and
generally no dose following procedure is required.
A seven-week-old baby is referred to A+E by her GP with a 2 week history of
wheeze and increased respiratory rate, poor feeding and weight loss. She was born at
term and was well for the first five weeks of life. There is no family history of note
and routine newborn examination had been normal.
On examination, she was tachypneic with subcostal and intercostal recession. She was
not clinically dehydrated. Occasional crepitations were heard in her chest. On
cardiovascular examination, she had a palpable thrill and a loud pansystolic murmur
heard best at the lower left sternal border. Her abdomen was soft and non-distended
but her liver was palpable 4 cm below the costal margin.
A chest X-ray was done which showed some patchiness in both lung fields.
She is admitted to the ward and reviewed by the cardiologists, who diagnose a large
VSD on echocardiography.
What should your first step in management be? Choose one answer only:
Do an ECG to ensure she is in sinus rhythm
Administer a loading dose of digoxin
Give a dose of furosemide
Ensure she is getting adequate feeds (at 150 ml/kg per day)
Start captopril
Give a fluid bolus of 20 ml/kg intravenously

Incorrect
Incorrect
Incorrect
Incorrect
Incorrect
Incorrect

Give a dose of furosemide


This baby has presented in heart failure resulting from a large ventricular septal
defect (VSD). These babies tend to present at around 6 weeks of age, once the
pulmonary vascular resistance has fallen, leading to an increased left to right
shunt through the defect and subsequent increased pulmonary blood flow (with
concomitant decreased systemic blood flow). The mainstay of treatment of heart
failure is diuretics and in this situation, with signs of respiratory distress and
pulmonary oedema on chest X-ray, you would want to give a dose of diuretics
immediately in order to get rid of some of the excess fluid. Furosemide is usually
used in this situation.

64

An ECG is of no use in this situation and may be normal or have no specific


features.
It is important not to fluid overload these babies and therefore a fluid bolus
would not be indicated, and feeds are often restricted initially as well.
Digoxin is not used in the acute treatment of heart failure. Captopril may be used
to try to improve ventricular function but likewise is not used in the acute
setting.

Incorrect
She settles fairly quickly with appropriate treatment and within a few days is no
longer tachypneic with no further signs of heart failure. Her liver is no longer palpable
below the costal margin and her feeds are able to be increased until she is demand
feeding well. She is discharged home after one week.
What would be the indications for early surgery (for example within the first few
months) in this baby? Choose three options only:
Failure to thrive
Ongoing heart failure
Persistent thrill and loud systolic murmur
Signs of pulmonary hypertension
Frequent episodes of upper respiratory tract infections
Cardiomegaly on Chest X-ray

Incorrect
Incorrect
Incorrect
Incorrect
Incorrect
Incorrect

Faliure to thrive
Ongoing heart failure
Signs of pulmonary hypertension
Indications for early surgical closure of VSDs include the presence of heart
failure, failure to thrive and signs of pulmonary hypertension. The presence of a
loud pulmonary component to the second heart sound and right ventricular
hypertrophy on ECG are of importance as they indicate the development of
pulmonary hypertension due to increased pulmonary blood flow from a large left
to right shunt. If left untreated, this could evolve into Eisenmengers syndrome
(cyanosis with reversal of the shunt from left to right to right to left), at which
point the VSD is no longer operable.
Surgical closure of VSDs is the commonest bypass operation performed in
infancy.

65

A 9-year-old boy is known to have a small ventricular septal defect (VSD). He


presents to A&E with a 1-week history of persistent fever and malaise.
On examination he is febrile and pale. His chest is clear and ENT examination is
normal. On cardiovascular examination he has a loud grade 3/6 pansystolic murmur
which is heard throughout the praecordium. There is also a 1/6 diastolic murmur
present which has not previously been documented in his hospital or clinic notes. On
examination his liver is palpable 3 cm below the costal margin and he has a palpable
spleen.
Routine bloods looking for signs of inflammation are requested as well a blood
cultures and a urine specimen. On dipstix his urine is positive for blood. There are no
organisms seen on microscopy.
A provisional diagnosis of bacterial endocarditis is made.
What are the most likely organisms causing the infection? Choose two answers
only:
Staphylococcus aureus
Candida albicans
Pseudomonas aeruginosa
Streptococcus pyogenes
Streptococcus viridans
Serratia marcescens

Incorrect
Incorrect
Incorrect
Incorrect
Incorrect
Incorrect

Staphylococcus aureus
Streptococcus viridans
Children with congenital heart disease are at risk of endocarditis, especially with
lesions causing turbulent blood flow such as a VSD, coarctation or in the
presence of prosthetic valves. Endocarditis should be considered in any child
with a known cardiac condition in the presence of persistent unexplained fever
or malaise. Other features include pallor, haematuria, splenomegaly,
arthritis/arthralgia, petechiae, Oslers nodes, Janeway lesions, splinter
haemorrhages etc. Further embolic events such as cerebral ischaemia, retinal
infacts and pulmonary emboli can occur.
The most common causes of endocarditis of those listed above are staphylococcus
aureus and streptococcus viridans.
Pseudomonas and Serratia are more common in intravenous drug users.
Candidal endocarditis is very rare.
In any case of suspected endocarditis it is important to have a definitive
66

diagnosis in the form of positive blood cultures and identification of the


organisms if possible prior to starting treatment. The treatment course is
prolonged, usually requiring 6 weeks of intravenous antibiotic treatment.

67

Potrebbero piacerti anche